100% found this document useful (1 vote)
521 views140 pages

Art of Problem Solving - Beast Academy 2A + 2B Books Set (4 - Jason Batterson and Erich Owen - San Diego, CA, 2019 - Art of Problem Solving - 9781934124314 - An

The document is a practice book for Beast Academy Guide 2A, published by AoPS Incorporated. It provides a series of math problems and exercises designed to reinforce concepts from the corresponding guide, including place value, addition, and comparisons. The book includes hints for challenging problems and instructions on how to use the practice book effectively.

Uploaded by

Anju Tandon
Copyright
© © All Rights Reserved
We take content rights seriously. If you suspect this is your content, claim it here.
Available Formats
Download as PDF, TXT or read online on Scribd
100% found this document useful (1 vote)
521 views140 pages

Art of Problem Solving - Beast Academy 2A + 2B Books Set (4 - Jason Batterson and Erich Owen - San Diego, CA, 2019 - Art of Problem Solving - 9781934124314 - An

The document is a practice book for Beast Academy Guide 2A, published by AoPS Incorporated. It provides a series of math problems and exercises designed to reinforce concepts from the corresponding guide, including place value, addition, and comparisons. The book includes hints for challenging problems and instructions on how to use the practice book effectively.

Uploaded by

Anju Tandon
Copyright
© © All Rights Reserved
We take content rights seriously. If you suspect this is your content, claim it here.
Available Formats
Download as PDF, TXT or read online on Scribd
You are on page 1/ 140

Digitized by the Internet Archive

in 2024

https://archive.org/details/beastacademypracOO000jaso
at.a7BT Bes de ws ad ths
b

aN
) 2017, 2018, 2019 AoPS Incorporated. All Rights Reserved.

‘eproduction of any portion of this book without the written permission of AoPS Incorporated is
trictly prohibited, except for fair use or other noncommercial uses as defined in sections 107
nd 108 of the U.S. Copyright Act.

ublished by: AoPS Incorporated


10865 Rancho Bernardo Rd Ste 100
San Diego, CA 92127-2102
info @ BeastAcademy.com

SBN: 978-1-934124-31-4
east Academy is a registered trademark of AoPS Incorporated.

/ritten by Jason Batterson, Kyle Guillet, and Chris Page


ook Design by Lisa T. Phan
ustrations by Erich Owen
irayscales by Greta Selman

isit the Beast Academy website at www.BeastAcademy.com.


isit the Art of Problem Solving website at www.artofproblemsolving.com.
rinted in the United States of America.
919 Printing

a
Se
oO}

. RO
Gj

Cy

oe

WO
NZ

GU

SR)
<7

We & oO

Cg
aX
2
Oy

¢ Place Value
« Comparing
« Addition i wo Odds |
Y x Z S A . . ( & vO

_ “oo - Ew

For more resources and information, visit BeastAcademy.com.


Each
This is Beast chapter of this
Academy Practice oF Practice book
MATH \ corresponds to
PRACTICE a chapter from —
Beast Academy
Guide 2A.

he first page of You may also read


chapter includes a the entire chapter in the
mended sequence for Guide before beginning
Guide and Practice the Practice chapter.
books.

Use this Practice book with


Guide 2A from BeastAcademy.com.

Recommended Sequence:
Use this Practice book with

mene an Book
Guide 24 from BeastAcademy.com

Pages
Eee Guide: Ue a
cona
petan Practice:
Guide: =)
24-29
Practice: 2-17
Guide: 30-39
Practice: 18-35

You may also read the entire chapter


in the Guide before beginning the
Practice chapter.
Some problems
in this book are very Every Yow to Use This Book
challenging. These problem marked
problems are marked with a %& has a
with a w. The hardest
problems have two Hints for
stars! the starred
ae : | problems begin }
Se eer the problems in the section.
\. on page 94. 5

tex! Ihalgosshere uaually includes an


explanation of the solution to the problem given
in the exarno ‘

PRACTICE | Instructions to the practice problems go here.

Since this problem has a star, it must be really hard! Problems with
stars have hints in the back of the book, starting on page 5.

After problem 54 Is problem 55. All of the problems in each chapter


are numbered in order, starting with problem number 1. Since this
problom has a pencil, your answer should include an explanation

rela

A complete
None of the Solutions.
explanation is
problems in are in the back,
given for every
this book require starting on
problem!
the use of a
calculator.
61. You mayboasking yoursoit, “Whyoidtheauthor
{90 10 80 much trouble writing text that no ona can
‘oad without superhuman Vision?” There's no
(9505 answor, realty,
answer, Sometimes, we wil provide a solution 62. Somotimos we give two possible ways lo solve a
‘hal in diforont than the way you solved tho problem, problom, Usually, those twowaysare Boparated by the
Wis usetut to read the solution to every problom even it era *OC ind wore Ines ee ihe ota below
yougotthe answers correct. You may loam 0 ferent
way lo 800 the problem, Or, you may find = mistako
thal wo mado! Wouldn't it bo Tun to waite tho authors to The second solution would como here. Generally, tho
pont out a mistake thay made uickar or more clever solution comea second,
55, Those aro just fake solutions that we wrote 60 that 83. Here is ancther fake solution with soma da
It looked lie there waa text hore, It wo put the real {rom a ditforont book in tha Beast Acadamy
solutions hare, I would give away some of the anawore Can you figure out what ia going en just by looking
tothe problems thatshow up in tho book. Then, you ‘tho diagrams?
‘wouldnt beable fohavefuntryingto salvethemon
your own! Hore are some fake diagrams to gowith
the take solutions
Oo Beh F >}
ce Sq ® BB BP
56, If you are having trouble with @ problem, It can be:
{amtng to ist ook in tho beck ofthe book forthe
tznewon Te more you bic about a problem, the Dolo ae lage VAD Ke se So
more you lam by song (hi ls usualy Vue even re 15
yeu get he wrong anaver), Do your beat gure
tule problem befor loking forthe anawer, Dont
forgo vee the hints forte stared probere bore
feeding he aon,
57. Hore aro some more diagrams to make these (ake Can you guess what the question that goes with the

jo 240 oqueree
solutions look like real ones: diagrams above waa? Here's a hint: The answer

ss 7 64, Thore's another hoader below,

a eee sreperene
Re Beinilets
58. some mathy stuff;
61 +62+63=80+60+50+142+3= 18040» 166 Zorpia Jonah veda placentae sto ie
65, The Dberish bolow is

rest of the apace below


Litalniaatel
‘bold, This makesatl it easier
Weeltofind thedeel
answer fo each 66. Curabitur
Ia saeras velit
Naan rial,Ouefata
eletend cals
neo interdum
as wine 1d,AT
commodo
Brae
59. The header below separates sections of problems. Voluipal. Aenean quis diam ut ligula volutpat dapibus,
‘Tris makes it easier to find tne solution you
«ro Jooking for. ee
67, Makes no sense does ii? | think some of those

ep ssseeuuyeorucyanen
essnatie
osmauuns cronsaunco
68. Morblrurum consoqual eos arm. Praesent
‘eget volt magna. Ouls votutpal auctor nequ
60. Did you notice the number on the right wide of the congue. Aliquam eral volulpal, Suspendisse potent
header above? Thal Is the page number where you can Pollentesque ut nulla lorem. Morbi consectetur purus
find the problem in the Practica book Thal way, when volutpat enim vestibulum moncus sodales ligula aliquet,
you are all finished reading # solution, you can fp back 69, Reedy 10 got started on some math problems? Get to it
to the page you were working on,

Protend Solutions | 111)


Use this Practice book with
Guide 2A from BeastAcademy.com.

Recommended Sequence:

Book Pages:
Guide: 14-23
Practice: 7-11
Guide: 24-29
Practice: 12-17
Guide: 30-39
Practice: 18-35

You may also read the entire


chapter in the Guide before
beginning the Practice chapter.
ee
£§SE*
sc ZgEL=
SSH
=Uco
4) ee
°c
ESM
o

S14ePWITN
(ab)

es
==
o50ne o>oeaL
SHegog
gm33952 oM
=
we Cc 5
=

Seu
Y
ca
@
2oOC

hes
&
O=
O cw
|
VY
eyfoeDE& UY
ot

fey)ge0 ceU - m ° yy

OO re)
.
3< oO

P2041.
) i0)))ore
oe
7)
:
"

oa =O
29
one
O's = >
Bae
we = onsN aOooSfe =
oO 35=. © Za=
0 5
=4O
=) ) ® CO90 fet 6
Oo NY
®©
:
Ss
. % &
A

& =

oi ; a E

3 C, :

Beast Academy Practice 2A


We also never
Pirates never use more
use more than nine
than nine ®’s to write a
X’s to write a number,
number, since ten e’s can be
since ten X’s can be
replaced with one X.
replaced with one C.

Write each pirate number below using ZD


?
PRACTI
Sea erewest pirate symbols possible. =

. 9000000000000 = J “ect

XXXKKXXXXKXXe
rae \aeil ee
=e

XXXKXKXKXKKKXKXXX
‘“ ee

eerie’f"**
>
COXKXKXXKXXKXXXX
apr ES
0000 ee

XKXXXKXKXXX
©0000 COCO O6OOOOO8O8®
a r !
S| ee ee,

8 |Guide Pages: 16-23


Beast Academy Practice 2A
Add the two pirate
numbers below.

We can
add pirate
numbers!

PRACTICE | Add each pair of pirate numbers below. ays dD

2X on! Wien hier ae


|
10. eee + eccce
Zo
ie KK KK Keo “+ XX Keoee =

12. a eeeee x Xe
+xXx =
a

Beast Academy Practice 2A


Remember fo write
your answers with
as few symbols as ~
you can.

PRACTICE Add the pirate numbers below. Write your answet .


using the fewest pirate symbols possible. 37
(i
Se ra
13.
eget <p RenaspbisleelSo, 5
ee es

Xe
+ XXXKXK _ Casas
14. XXXXKeoe

ee:

15:
ee ee
Pep Ae!

16. XX Keooeee + KK KKK Koooce - C

AZ. Captain Kraken has X&XXX X coins. 17. 0a


He then discovers KX XK KKK
more coins. How many coins does Captain
Kraken have now?

18. Barnacle Barney buries two treasures. One


has U ¥¢,eeee@@ coins. The other has
X Xecccdeo coins. How many total
coins are buried? e
I ZS ps
L é

Guide Pages: 16-23 Ay Beast Academy Practice 2A


PRACTICE Write each answer below as a pirate number.

3S
19. Captain Kraken has X& XX@@@ coins.
He spends XX @ of the coins to repair
the sails on his s , How many coins does
Captain Kraken have left?

1b
+
20. A treasure of X& XX XK eeee0ee coins is
split into two piles. If both piles contain the

=
iN
<a \V
ie

Zils A treasure chest contains three bags of


coins. If each bag contains X Keeee
coins, what is the total number of coins in
the treasure chest? tg

WA
4 ~ af
Lh
Captain Kraken takes X&X@@@@ o
coins
from a chest of X KX XK X XX coins.
How many coins are left in the chest?

Beast Academy Practice 2A Za Guide Pages: 16-23 |11|


_
Digit: Stan te fee
a
If you’re not
2
X's, anderU'
e’s, it s to
wr e numb s.
0, 1, 2, 3, 4 5,
6, 7, 8, and 9 are :
the ten symbols that we
use to write numbers. s
These are called ae,
digits.

son
WN
oe
ee
For example,
43 is a two-digit
number that uses
the digits 4 and 3.

PRACTICE |Write each pirate number below as a two-digit number.

23. Xeee =
aaa 24. Keecscecsee = rb
ace

Sextewee 2 A ean cassie KERN Rea


i

XXXKXXXeeooee - Sf 28. XXX Xeeee - 45


peer

XXeee- 2° 30. XKXXX = ae


V\ ya
12 |Guide Pages: 24-29 .
ae
j Beast Academy Practice 2A
hwo five
hundveds tens
he

In a three-digit
In a pirate number, \ number, the digits tell
the U’s, X’s, and @’s us how many hundreds,
let us Know how many fens, and ones there
hundreds, tens, and are.
ones there are.

PRACTICE | Write each pirate number as a three-digit number.

rs

31. CK
XK XK Keee - | F% 32. CKeeccecee - [|

aid ee

33. COG xe 27722) 34. UX Keeee = /cf

35. CUOCCOXee = 5/ 2 36. Ceescee = (05


oo 7
37. CXXX = 130 [roo
8, CRCCELE =
ee
x8
b)e—~—

Beast Academy Practice 2A


me Guide Pages: 24-29 |13|
: place.
In 309, we oe
say that 3 is the | roe ee
hundreds digit, 0 is ~S Ss Ss
the tens digit, and 9
is the ones digit.

PRACTICE | Answer each question below about place value.

Circle every two-digit number below.


_

(55) 95 565 52

te
om

209 Se 7290 920

Circle every number below that has tens digit 7.

(477) a7 714 447 474, 4d

Circle every number below whose hundreds digit is larger than its ones digit.

978 798 879 897 “789 987

Beast Academy Practice 2A


DIGieS :
We never write 0
as the leftmost digit
of a number with more
than one digit. For example,
we always write
7 tens and 3 ones
as 73, never as
073.

PRACTICE Answer each question below about place value.

43. Arrange the digits in 322 to write a new three-digit number 43._232
that has a 3 in the tens place.

44. Arrange the digits in 750 to write a new three-digit number


that has ones digit 7.

45. How many different two-digit numbers have 0 as a digit?

Write three different three-digit numbers 46. 7%


that use the digits 7, 8, and 8.

Use the digits 5, 7, and 9 to write a number whose ones digit


is larger than its tens digit, but smaller than its hundreds digit.

Beast Academy Practice 2A


In he row ofdigitsbelo circle two iff ent
| en| — oe numbers that have 6oythe t ns ple ce .

> ferentSi :
: Belowistheonlyway to circletwo

Solve each Number Search below by


PRACTICE
circling numbers that do not overlap.

48. Circle a 3-digit number with ones digit 2 and hundreds digit 4.

23424Be)

49. Circle two different 3-digit numbers that have a 5 inthe tens place.

(5 6 (4\5\5\4 5) 5
ee ha . vi :

50. Circle two different 3-digit numbers that have a 9 in the ones place.

9 99/8 99)9

91. Circle two different 3-digit numbers that have a 0 in the ones place.

001 (0 o)f 10

Beast Academy Practice 2A


Remember, Lmbeh, Search :
the numbers
you circle
Passio
in these

Solve each Number Search below by


PRACTICE circling numbers that do not overlap.__

52. Circle three different 3-digit numbers that have a 0 in the ones place.

CC ad0@Ko120 Y¥

53. Circle three copies of the same 3-digit number.


ee
1261 4\21861 41261 4)2

54. Circle three 3-digit numbers that have their largest digit in the tens place.

87657 6)5 46 5)4(35 43 2 ea

Circle four 3-digit numbers thatall have the same tens digit.

Nec ig BG?
tf ;
Coa Ue A sold <

2425
x Ti) seeps
56. Circle four different 3-digit numbers.
*
(7Js373\3773373373
fe -
Xx
A
te, = mM

Beast Academy Practice 2A


7 Z A S LSE LISS OE NY SEP SNE LOT AR TIES

y Be

oY
18 Beast Academy Practice 2A
We can For these
combine 10 ones problems, you need
~ to make 1 ten, or to arrange some
10 tens to make blocks into larger
1 hundred. groups.

PRACTICE

59.

61. 62.

fe
61. b 62. IGA

63.

Beast Academy Practice 2A


PRACTICE |Answer each question below.

64. What two-digit number is the same as 7 tens and 17 ones? 64.

65. What two-digit number is the same as 3 tens and 28 ones? 65.

66. What three-digit number is the same as 5 hundreds, 2 tens, 66.


and 11 ones?

67. What three-digit number is the same as 15 tens? 67.

Beast Academy Practice 2A


68. 5 tens and 26 ones is the same as__6__tensand_1|€__ ones.

69. 5 tens and 26 ones is the same as 7 tensand © _ ones.

70. 7 tens and 14 ones is the same as __8 _ tens and 4 ones.

71. 7 tens and 14 ones is the same as_6 _ tens and 2 4 ones.

Try these last


three without
using blocks.
12 tens is the same as ___! hundred and _2 __ tens.

3 hundreds and 7 tens is the same as __3/ tens.

222 is the same as 2! tens and __12 __ ones.

Beast Academy Practice 2A


Kegrouping

This chart shows


five different ways
to group 74 into tens
and ones.
What is the
missing number
in this chart? _/

n break a ten.

Fill the missing entries in each chart below so that


PRACTICE each row describes the number of blocks to its right.

» Beast Academy Practice 2A


Fill the missing entries in each chart below so that
Fauaes each row describes the amount above the chart.

TAG 78. 79.

80. 81. 82.

Vv oS

83. Fill each blank. 84. Fill each blank with a 3-digit number.

Bq ie 17tens = Ii?- o a
890 = tens

605 = 60 tens and 5 ones 90 tens = 4 0 a

534 = 50 tens and 34 ones 4 hundreds and 44 tens = _ 4446


“4
GO: x
da 4h ©
oe |23|
Beast Academy Practice 2A yee
We can use 2764
what we Know EXAMPLE |Add: 276+10
about place value to
help us add one, ten,
27641000
or one hundred to a
number. 276+1: Adding 1 to 276 increases its ones _ : : .
digit by 1.S0,276+1=277.
276+10: Adding 10 to 276 increases itstens
digit by 1. So, 276+10
= 286.
<2 276+100: Adding 100 to 276 increases its hundreds [s
digit by 1. So, 276+100=376. —

PRACTICE | Solve each addition problem below.


ee
cL ae ; x
Sng aps 6. 134+10-|44 +97. 134+100= 12

+6 es > A ;
noise al = vad g9. 555+10- 709 ‘go, 555+100=-
Gh

x a
91. 7e8+1=26 7 “ go zegsio= 179 93. 768+100= $&f
a -

94. 987+11= gg Ons 798+101 = 644 face 879+110= TP)

va

ei Urs “
97. 7294+11= qh 0 “98. 279+101= ps 99. 792+110= 40 )
*

Guide Pages: 30-39 Beast Academy Practice 2A


: Wtndreds
XAMPLE ‘Subtract: 427 -10
A27- 100 We can also
subtract 1, 10, or 100
i Shoratoast1 446m 427 decreases its© from a number using
what we Know about
: ceonesa) BE So,AT 1 = 426. place value.

pentaea digitby1— 427- 100=


alae

PRACTICE |Solve each subtraction problem below.

100. 615-1= 614 101. 615-10= 4095 102. 615-100=_5/f

103. 888-1=_ ZY/ 104. 888-10= 2 ay 105. 888-100=


1%

: 1-5-
106-2432 _ -1 = 4Z,
( we 107. 432-10
_ = 4h
ZO )
108. : 432-100=
a =
337Ss)

109. 543-11=_ 922 110. 354-101= <9} 111. 435-110=_325

112. 120-1= BY 113. 102-100=__ <= 114. 201-10=_ 2G |

Beast Academy Practice 2A Guide Pages: 30-39 |25|


(Ones, Tens, and Hundreds —

PRACTICE | Fill the blanks to complete each pattern below.

Mo. OC TO< eon one a Geetag vlog, jqseraaiG


CN OS
115, 2.130.140 «» 150 (ot
wsES pe ae UR SS LAY f° Mali pa oS EE SS ee
270Sy, }

+10 +10 +10 +10 a0 +10 +10 +10


Yee MY oe | ee
Ge cDD 26a, 275 >Z£ 24 Sp) Saed /&

el Oe Oe Oe AOr DOr” ent e) +16


NE ed eS
117. ee A yn ee ee) 2 eee eee ee ee ie SS Sen es S 2 “ae

+10 +10 +10


+0 J +10 +10
+10
ee aaa | TH 44 434 44
18, D44354 Wd Be DO
ae. ae =
|26 Guide Pages: 30-39 Beast Academy Practice 2A
PRACTICE Answer each question below.

119. Fill each blank with the correct answer:

800-10= 703-10=

120. There are 506 chipskunks in a field. After 10 chipskunks


run away, how many chipskunks are left in the field?

121. Captain Kraken’s secret treasure is 405 lollipops. 121.


His crew eats 10 of them. How many lollipops are
left in his treasure?

122. Grogg’s number is 1 less than Lizzie’s.


* — Lizzie’s number is 10 less than Alex’s.
Alex’s number is 100 less than Winnie’s.
Winnie’s number is 209. What is Grogg’s number?

Beast Academy Practice 2A poo! Guide Pages 30-39


In a Digit Difference Grid, numbers in squares that share a side must.
ad
be 1, 10, or 100 apart. For example, a square next to 355 can be fille
with any of the six bold numbers below. ~

355+1 = 356 | a05e 1=354


365+10= 965 355-10 =345
355+100=455 —«-355-100=255_
~ Anumber can only be used once in aDigit Difference Grid.

Fill the empty square to complete the


EXAMPLE following Digit Difference Grid.

2/51
Spb
eourrayyziC
ts get from 354 to 255, we subtract 100 and add 1,
or add 1 and subtract 100.

So, the nore in the empty square is either 354-100= 254,


or 354+1 =355.
Each number can only be used once. 355 is pean |
in the
_ grid, so the empty square must be 254.

Fill the empty squares to complete each


PRACTICE
Digit Difference Grid below.

’ Lyi

pe 124.6 ) 2c}566] 15 [158]


7]175
x } |
Guide Pages: 30-39 Beast Academy Practice 2A
Fill the empty squares to complete each
FENG: Digit Difference Grid below.

«(es
129.

131.

Remember,
you can only
use a number
once in each
Digit Difference
Grid.

134.

Beast Academy Practice 2A


eae
<<
EN

a
<7

Fill the empty squares to complete each


PRACTICE
Digit Difference Grid below.

“le} Le]

l=] = [mle] fl
138. 139. 179] 140. 199

141. Bos 142. 67 143. |285 |286 |296


Bo o1
pa
oO)
EE feel
|30| Guide Pages: 30-39 Beast Academy Practice 2A
Fill the empty squares to complete each
PRACTICE
Digit Difference Grid below.

149%6 47.9 16972962

173
eT |e

aac

Pace
fel
*

Beast Academy Practice 2A


We can add EXAMPLE Add 427+60.
or subtract more
than 1 one, 1 fen, at 0is 6 tens. So, adding 60
6 is
s thesame as
or 1 hundred at
a adding 6 tens.
427 is 4 hundreds, 2 tens, and7 nee
Adding 6 tens to 427 gives
4 hundreds, 8 tens, and 7 ones, which is 487.
Adding 6 tens toa number increases its tens
digit by 6, unless we have to regroup.

PRACTICE | Answer each addition and subtraction problem below._

153. 213+500=_[[> 154. 21345=4!f 155. 213+50=


25 >

156. sas-4= D)) 157. 586-400= |)“ 158. 586-40= 990

159. 121+70= 19) = 160. 1214+7=1


25 — 161. 121+700=_

162. 475-300= |14 163. 475-30=_ 44 164. porary)

Ea Guide Pages: 30-39 Beast Academy Practice 2,


PRACTICE Answer each addition and subtraction proble'
below. Write your answers using digits.”
(NM,4

165. Add:

166. Subtract: 166. 44 His. 7h

Ze

167. Add KX KM Kee + MRS K Keoooe 167. 42+64=/0)

168. CUKee —
Subtract KK KK 168. 112-40= 67

169. Add 7 tens to 683. 169. 683+70= 223 __

170. Subtract 8 tens from 446. 170. 446-806= —3bG

Beast Academy Practice 2A


PRACTICE |Answer each problem below.

171. What number is 111 more than 789?

172. Write the four different three-digit numbers that use the
digits 0, 1, and 2 once each.

173. How many symbols are in the pirate number for 789? 173. _+ 2
Review pirate numbers on page 7.

Dec
be
BLXxXdx
CECCO
(aes
174. There are only three 3-digit numbers that can be written using only
* two pirate symbols. Write them as pirate numbers.
Review pirate numbers on page 7.

“ihe, OX
— i
c —~
tt -

Beast Academy Practice 2A


PRACTICE |Answer each problem below.

175. Fill the blanks to complete the pattern below. es


i ae ee a
30 w80 «450: “430 oe0m ee cone cone mee
i NS NSE Nee Ly

ee

176: What three-digit number is 77 tens plus 77 ones? 176. K4I

at

177. Adding a number's digits gives you the number’s digit sum. 177. [2-9
For example, 345 has a digit sum of 3+4+5= 12. What is the i
smallest three-digit number that has a digit sum of 12?

178. lf you write every number from 1 to 100, how many times 178. /[b
will you write the digit 5?
aad

Beast Academy Practice 2A |35|


Use this Practice book with
Guide 2A from BeastAcademy.com.

Recommended Sequence:

Book Pages:
Guide: 42-55
Practice: 37-49
Guide: 56-67
Practice: 50-63

You may also read the entire


chapter in the Guide before
beginning the Practice chapter.
_ A number line
_ shows numbers in
order from left to
right.

— marks on ©
‘the number line
tand for whole
numbers.

Label the missing numbers on


Ju MANGS each number line below.

Beast Academy Practice 2A Guide Pages: 42-47


Label 50, 60,and 63 on
EXAMPLE |the number linebelow.

The large tick marks


between 40 and 70 are
50 and 60.
Then, 63 ts
| the third tick
\mark to the right /—
of 60.

Draw arrows as shown in the example


FehOUGS above to label the numbers given.

5. Label 16, 20, 25, and 43.

fpf tent intntanbenfenenfenpeape chapel etm ah af


10 50

6. Label 61, 75, 84, and 90.

} 100

7. Label 34, 45, 56, and 67.

|38 Guide Pages: 42-47 Beast Academy Practice 2A


Number lines
don’t always
i have a tick mark
for every whole
number _/

PRACTICE |Answer each number line question below.

8. Label the missing numbers in the boxes below.

80 100 150

9. Label the missing numbers in the boxes below.

320 380

10. Label the missing numbers in the boxes below.

150 300

11. Label each arrow with a three-digit number that includes the digits
*& 2 4, and6 once each.

a
Beast Academy Practice 2A Guide Pages: 42-47
v 7 How far is 37 from 62 on oo / We can find the
@\ EXAMPLE © the number line? , distance between
- : two numbers by
Ae +10 : counting up on the ,
, * ON at Ne number line.

Y 40° 60 60}
37 - [:
: From 37 to AO ij
is 3 units. ;
From 40 to 60 is 10+10=20 units. :
| From 60 to 62 iso units.

: 7 So, the distance from o7 oO62 is


— 3+20+2=25 units.

PRACTICE | Find the distance between each pair of numbers below.

12. How far is 19 from 60 on the number line? 12.

20 60

13. What is the distance between the two dots on the number aoe
line below?

Opp pee ecpte penta feibjt ache apptm peel ne haftle


90 100 TO 120

14. Find the distance from 54 to 81 on the number line. 14.

SS
L aOL O
40 80

Guide Pages: 48-54 Beast Academy Practice 2A


Fillin the blank: 58 and ___ are the same Distance
ee distance from 70 on the number line.

The distance from 58 to 70 is 2+10 = 12 units. We say that 70 is


—. oe — halfway between 58
and 82.

— 80

To find the other number that is 12 units from 70,


~ we count up 12 units from 70.

ee 70 go }
0 _ | a
IG

| | So, 58 and 82 are the same distance from 70.

PRACTICE | Fill each blank below.

15. 18and are the same distance from 25 on the number line.

Se
18 25

16. 33 and are the same distance from 50 on the number line.

SS pea oyVe Lag a


33 50

17. 98 and are the same distance from 73 on the number line.

EEE EEE EE
73 98

Beast Academy Practice 2A Guide Pages: 48-54


EXAMPLE |What number is halfway between oa 55? :
ing

same =
We can count in from 15 arid 55; As long as wes count in Bythes
amount, the number halfway between our points stays the same.
+5 — =o

The number halfway between 15 and 55 is the same as


UB207228g
£/0fK
AO the number halfway between 20 and 50.

Weeen counting in a equal amounts unt we reach the middle.

2s +10_ bee Aiiee ge -5

So, 35is halfway between 15 and 55.

PRACTICE | Answer each question below.

18. What number is halfway between 45 and 65? 18.

a a SSA AL SLE RE SE RE Re teed


45 65

19. What number is halfway between 210 and 260? 19.

210 260

20. What number is halfway between 27 and 73? 20.

LAS MARARRRREE ORBRRREEES EBEREEEEEE EEnGEnnnae neem cee


27 73
Guide Pages: 48-54 Beast Academy Practice 2A
Between -

PRACTICE | Answer each question below.


)

a What number is halfway between 32 and 76? 21.

A | ! A !
32 76

oa What number is halfway between 534 and 584? 22.

A | ! A
934 584

Try solving the


problems below
withouta number -
tine!

23. What number is halfway between 70 and 130? 23.

24. What number is halfway between 118 and 162? 24.

25. What number is halfway between 43 and 89? 25.


*

26. Is 64 closer to 40 or to 90? 26.


*

27. |s 385 closer to 350 or to 410? 27.


*

Beast Academy Practice 2A Guide Pages: 48-54


Number Line Conquest is a game for two players.
Start with an unlabeled number line. Players take turnslabeling tickmarks ¢as :
“bases” until each player has threebases.
After all six bases are labeled, the game is scored. Players get 1 oe
for each blank tick Bark that is closer to one of their bases than to their
opponent's.
The player with the most points wins.

Who wins the game of Number Line Conquest


_ EXAMPLE
between Winnie (WW) and Grogg (G) below?
G

nna
(7
Aen
Pe, Winnie labels saci)blank tick mark that is closer to one of her bases with a w.
| Grogg labels each blank tick mark that is closer to one of his bases with a g.
The tick marks that are the same distance from both players’ bases are not
scored for either player.
| WW wo WwW g oe 4 7. :gq o WW gg

Ww GW “Gar ING
Winniescores 6 as and
aor scores 5. So, Winnie wins.

Find a partner and play Number Line Conquest on


PRACTICE
the number lines below. Take turns going first.

Print more game boards at BeastAcademy.com.

Beast Academy Practice 2A


Place a third X on each game board below so that
PRACTICE
Player X wins the game of Number Line Conquest.

28.

30.

gos X O

Beast Academy Practice 2A


In a Honeycomb Path puzzle, the goal is to fill every empty hexagon (©) with
a number so that a path of consecutive numbers crosses every hexagon.
Fill the empty hexagons to solve the
: EXAMPLE Honeycomb Path puzzle below. | _ Consecutive
numbers come one
after another. For
example, 11, 12, and 13
\. are consecutive.

SYOCDNPUCLK
SY
29d
In this example, we
fill the empty hexagons
with 13, 15, and 18 to
complete the path!

Fill the empty hexagons with numbers to


Buus solve each Honeycomb Path puzzle below.

33.

Beast Academy Practice 2A


neycond Paths

Fill the empty hexagons with numbers to


SSS solve each Honeycomb Path puzzle below.

37.

Beast Academy Practice 2A 47


~

aN

Fill the empty hexagons with numbers to


PRACTICE
solve each Honeycomb Path puzzle below.

Beast Academy Practice 2A


S

PRACTICE Fill the empty hexagons with numbers to


solve each Honeycomb Path puzzle below.

Print more of these puzzles at BeastAcademy.com.

Beast Academy Practice 2A


An equals sign (= )shows that two amounts are | We have
the same value. For oes symbols to.show
when one value
is larger than ,
another.
If two errounts areeho: Saiswe use < OF >
_ show which is larger. The < and >
symbols always
The < symbol means “ig pee than.” “eat” the bigger
The > symbol means “is greater than.” — amount!

7 i read “Sixisless than seven,” and


100 >99 is read “100 is greater than 99.”

CC
Neen”
~ a

= 5 (52.

53. 54.

D0: 56.

57. 19 ones (S) 2 tens 58.

59. 199+202 6) 200+200 60.

Guide Pages: 56-59 Beast Academy Practice 2A


Fill each
blank below
with a digit.
For example,
[5|8 is the
number 58.

PRACTICE | Answer each question below.

61. Fill the blank with a digit to make the comparison below true. VAY,

6|0|<61 ey,
62. Fill the blank with a digit to make the comparison below true.

7|4>85
63. Use the digits 2, 3, and 4 once each to make all three comparisons true.

30<|#|1
7\|#|>73
2|3<34
64. Use the digits 5, 6, 7, and 8 once each to make all three comparisons true.
*
7|1>62

Guide Pages: 56-59 |51|


Beast Academy Practice 2A
\
RS
Do
CS
ae

2
ye
U7

PRACTICE |Solve each problem below.

65. Order the numbers 32, 233, 323, and 223 from least to greatest.

66. Order the numbers 714, 471, 741, and 417 from least to greatest.

67. What is the greatest three-digit number whose digits are 67.
all different?

68. What is the smallest three-digit number whose digits are 68.
all different?
Guide Pages: 60-67
Beast Academy Practice 2A
| EXAMPLE Order the numbers below
~ | from greatest to least.
/ When comparing
more than a few “87, 11).203,
96 451, 8 112
numbers, it helps to
organize them by
\ place value.
Hundreds
: eieernhundreds digit,followed by 203,
matter more
| than tens, and tens _ then by 112. So, the first three numbers
matter more than in our list;
are 451, 203, and 112.
d}96are the only remaining
th a tens digit. From greatest
ave 96, 87, and 41.

| The smallest number is 8.

| So,fromgreatest to eae we have


7 Abi, 208, 112, 96, 87,11; 8.

In the problems below, fill the blanks to


BILAN order the numbers from greatest to least.

69. #5 P 70.45 6 71. i) 263 «© reatest


id _v 34g) 11M), 3 &
ae 435 f 919
a3 54 IT gO +t
6 | OR ito 99
76a | 543 149} 75
a ca 5 IG g| OO
2 in en 19) 74 Least

Beast Academy Practice 2A Guide Pages: 60- or [53]


In a Number Path puzzle, the goal is to trace the paththat
crosses alof
th
numbers in the grid from least to oe - —

| Trace eee that Sano the numbers inthe __


EXAMPLE grid below in order from leastto greatest. —

[|] sar
mc
[shes
resize
We start at the
smallest number, “
92... ..then connect
numbers from least
to greatest until the path
has crossed every
number.

In each puzzle below, trace the path that connects


PRACTICE
the numbers in the grid in order from least to greatest.

54 | Guide Pages: 60-67 Beast Academy Practice 2A


In each puzzle below, trace the path that connects
ee the numbers in the grid in order from least to greatest.

74.

peak (M705a
=: bg anal (
Cae

TT.
ie

79.

78.

Guide Pages: 60-67 |55|


Beast Academy Practice 2A
We can use
_< and > symbols to
compare more than
two numbers.
.

PRACTICE |Solve each problem below.

80. Use the blanks to order the following numbers from least to greatest.

12% a 16 108 61 18

81. Use the blanks to order the following numbers from greatest to least.

65 506 650 560 605 96

82. How many different whole numbers could replace the, gray 82.
box below to make a true statement?

10<f<20
[56| Guide Pages: 60-67
Beast Academy Practice 2A
PRACTICE | Solve each problem below.

83. The eight numbers below use only the digits 2 and 9. Use the blanks to order
these numbers from least to greatest.

29 99 222 9 22 Z 92 229

84. Fill each blank below with a number using only the digits 3 and 4.

Gik CS 4 ore 44 < 334 < < —74305

85. There are six different numbers that can be made using the digits 1, 2, and 3
* exactly once each. Two of the numbers are shown below. Fill the remaining
four blanks so that all numbers are in order from least to greatest.

23a < < < 312<

86. The statement below lists the smallest eight numbers that can be written
* using only the digits 0 and 5. Fill in the four missing numbers.

0<5<50< < < < < 555

87. The statement below lists the smallest eight numbers that can be written
* using only the digits 7 and 8. Fill in the four missing numbers.

9 Pn 5 i he < < < < 778

Beast Academy Practice 2A Guide Pages: 60-67


~~
ae

EXAMPLE

NS

The first and


last number each
have tens digit

-___{ middle number


also has fens

We fill the
remaining blanks
LQTS

with 6 and 8.

In each problem below, use the given


RRAGTIGE digits once each to fill the blanks.

88. Digits: 4, 7,8 89. Digits: 3,5, 9

2<(7|2<|2I <| |O<| |0<80

90. Digits:
2, 4,6 91. Digits:
4, 5, 6,7

34<3] |<4| |<4| | |


|<|<[
4[ ]4<60
|58| Guide Pages: 60-67
Beast Academy Practice 2A
)

In each problem below, use the given


BupUey digits once each to fill the blanks.

92. Digits: 6, 8, 9
‘It might help to
9<9| |<| |7 cross out the digits
as you use them! —

93. Digits: 5, 6, 7

5<| |<| |5<| |5<75

94. Digits: 1, 1, 1, 1, 2,2, 2,2

LT<LU<LLI<L
Digits: 3, 4, 5, 5, 9,9

o<L|_I<_l_J<LL|<70
Digits:.5;.5;55,-5; 7,./9;979

[<L7<LL<7L<Li7<_Lu
Beast Academy Practice 2A Guide Pages: 60-67
>

Ne

PRACTICE | Answer each question below.

97. What number is halfway between 7


* 123 and 321?

98. Label the missing numbers in the boxes below.


*

15 60

ae Alex, Lizzie, and Grogg each pick a number on the number 99.
line. Lizzie’s number is halfway between Alex’s number and
Grogg’s number. If Alex picked 25 and Lizzie picked 55,
what number did Grogg pick?

Beast Academy Practice 2A


PRACTICE |Answer each question below.

100. Ms. Q marks the number 50 on the number line. 100.H1 j GB


*® Alex marks the number 15 units to the left of Ms. Q’s number.
Grogg marks the number 25 units to the right of Alex’s number.
Lizzie marks the number 35 units to the left of Grogg’s number.
Winnie marks the number 45 units to the right of Lizzie’s number.
What number does Winnie mark?

Sr

101. In the mon, mi dhideachtla tre stands for a digit, 101.


* and different shapes stand for different digits.
cept

What digit does each shape stand for?

Beast Academy Practice 2A ;


PRACTICE |Answer each question below.

102. Alba fills every blank below with the same digit to make 102.
*® the comparison true. What digit does Alba use?

| |7[ l<[| |Jo<g9{jo

103. Tia’s favorite number is between 777 and 999. 103.


* The ones digit is less than the tens digit.
The hundreds digit is less than the ones digit.
What is Tia’s favorite number?
core

104. Winnie arranges seven whole numbers in order, then covers the five numbers
* in the middle with shape cards. List all of the numbers that could be hidden
under the card.

15<[@] [a |<[]e[m]<[025
104.

62 |
Beast Academy Practice 2A
Problems

PRACTICE Answer each question below.

105. Fill the blanks below using only the digits 1, 3, and 6. Some of these digits
* may be used more than once. J

is halfway between nn and Gili

106. Each incorrect statement below can be corrected by swapping two of the
* digits. Swap two digits in each statement to make it correct.

7\<[6l6)<5\5)* —_[siisi<(2ll9/<(3/2 *
<Zlal<Glgl
2s y 2lsl<4)<Bl2) v

¥

f2|<[12i<i2}* — al2}<(5[5| A 2|2| *


Aizi<ii2isidlv = [dai<l2ii A pll2 ¥
-} —y ~
1 f f
| : ¢
1

Beast Academy Practice 2A


Use this Practice book with
Guide 2A from BeastAcademy.com.

Recommended Sequence:

Book Pages:
Guide: 70-75
Practice: 65-69
Guide: 76-87
Practice: 70-85
Guide: 88-93
Practice: 86-93

You may also read the entire


chapter in the Guide before
beginning the Practice chapter.
EXAMPLE | Add 25434. To find
| The result of S on. a a sum, we
addition is called ) 2S 2 tens and 5 ones. can add the
a sum. ' 34 is 3 tensand 4 ones place values
separately.

PRACTICE | Fill the empty blanks to find each sum below.

t -32+56==80- + 8 = $7 2: oot G0 + 4 = 4

3. 62+34= 90 fe 4¢ 4. 54421=- 70+ 5. ‘


5. 217+461= 600 + Geb is 6. 47+32- 79_

7. 61425= pe 8. 136+53= [ff 9, 644+315= 9/9

10. 250+37-2L/ 11. 270+604-$/4 12. 316+260= _G7¢

Beast Academy Practice 2A Guide Pages: 70-75


EXAMPLE Add 73+84,_

| 73 is 7 tensBAG3 ones.
We can
only have 84 is 8 tens and 4 ones.
one digit in
So, 5, adding 73484 gives : /10 ones make
each place
value. sometimes \ 7+8= 15 tens and 3+4=7 ones. 4 1ten, and
we need to 10 tens make —
regroup. , 45 tens is the same as 1hundred : 4 hundred.
. ae esor 150. eT Pere

PRACTICE | Fill the empty blanks to find each sum below.

1S 04 Oo=— ie0™ + 6. f= 2 14: 2446S) Sar oe

15, 04419= 60 +s)


- =.) 16 86428 =—502 + — Pee ee

Wife SBS ee eA)a 18:- S9*39


80 +2 oS
= ss

19, > +.
52+81=. =. 920) 67¢26e.0 ee ee

D1 S8H65=
we eee 22. 85+65= +s

66 Guide Pages: 70-75 Beast Academy Practice 2A


PRACTICE |Fill the empty blanks to find each sum below.

2eaececotgo—1300ee+
. 110) + 8" =

24, 7 52604540 — *800te+ = 60 + =. =

255 COO F460 =) 700: +. 3 4

26. S15th0e
eee cee Pass

27. 248+590= Meee) + =

28 (2/3204-= aa te tS ee

29. 883+109= someon Selgin fig Pe caaceoms

30. 555+346= a PCs Beg eee ae Ie

Guide Pages: 70-75


Beast Academy Practice 2A
Fill the empty
Complete the Cross-Number :
boxes so that the EXAMPLE
sums from left-to-right puzzle below.
\ and from top-to-bottom
are all correct. Across: Down:

=e [20]+34=54. ‘[20]+21 =[41]


at} + [23] =) a = aa
--B

The completed -
Cross-Number
puzzle is shown
_on theright.

Find more Cross-Number puzzles at


BeastAcademy.com.

Beast Academy Practice 2A


212

Beast Academy Practice 2A Guide Pages: 70-75


Thinking
about addition
on the number
line can make
some problems
easier.

PRACTICE |Fill the empty blanks on the number lines below to find each sum.

39.

196 200 Fost |

40. 880+88 |

Guide Pages: 76-81 Beast Academy Practice 2A


a1. 950457 |
Fab
+
0. enamel
350 400 ee

olan Poe
q

. pie
43. 391+39 4 ii

391 400

44. 193+22= 45. 496+56= 46. 598+223=

Beast Academy Practice 2A Guide Pages: 76-81 | 7]


eee Ss basket contains
EXAMPLE"| balls. Marjorie’ s baske’
45 tennis balls. How man\
7 balls are inboth baskets’
maine tennis balls ae one basket to the other
does not change the total number of balls.

(swig
421907 To make the addition easier, we.take one.nak
oo from Marjorie’ S basket and putitinigors.
a So,|instead ofpacing49+45, we add 50+44.

PRACTICE |Fill the blanks to find each sum below.

47. 39+26=| 40 |+| |=| 48. 200+86=|300|+| |=|

49. s2+i28=| |+(130|=| 50. ve+96=| |+{100|=| |

oI).
serie] 92 seseeer-[ HL
Guide Pages: 76-81 Beast Academy Practice 2A
PRACTICE | Answer each question below.

53. Marie has two pitchers, with 96 ounces of water in each 53.
pitcher. She pours 4 ounces from one pitcher into the other.
How many total ounces of water are in the two pitchers?

54. A parking garage has a lower lot and an upper lot. There 54.
are 277 cars in the lower lot, and 203 cars in the upper lot.
After three cars drive from the upper lot to the lower lot,
how many cars are there in the whole garage?

55. A bookcase has two shelves. There are 93 books on the 55.
top shelf and 78 books on the bottom shelf. How many
books could you move from the bottom shelf to the top
shelf to make it easier to find the total number of books?

56. Sander has 98 peas on his plate. Ada and Parker each 56.
have 101 peas on their plates. What is the total number of
peas on all three plates?

Beast Academy Practice 2A Guide Pages: 76-81


When adding,
sometimes it’s useful
to add a little extra,
CIES
Cc
NZS
Av

< yo é

57. Adding 38 to a number is the same as adding 40, then taking away

58. Adding 19 to a number is the same as adding , then taking away 1.

59. Adding to a number is the same as adding 100, then taking away 7.

60. 133+28=| 133 |+| 30 || |

61. 216+91 -| 216 || |] |

62. 256+95
27 ee ee
74 | Guide Pages: 82-87 Beast Academy Practice 2A
TaSZ

< Ls
MSZ

aS
DRO

PRACTICE Fill the blanks to answer each question below.

63. 27+27+27=| 30|+|30|+(30] =| |

64. 99+99+99+99+99= 500|-| |

es. 298+200+200-[
+3 ]<[]
66. 254+244+234+22 =|100|-|
|= tf
67. 9+194+29+39
ceca Lace
68 190+190+190+190
aa aoe
Beast Academy Pract Ice 2A Guide Pages 82- 87 75
There’s
more than one
way to find a
sum. |

PRACTICE |Find each sum below.

69a s98+96=.. 70. 589706= er Oe

72. 538+140= ————i7«’WsdW'4O=——<“<t—séiHCSSSC«295 +14 =

75. 713+65=_——“<ité‘iGWSd18354+65=_——————~S~Szzw’s«O470
BH =

7B: BBOdAddS wee 879, 6299+: gorsier eae

81. 75+426= (si. O74 75= 83. 680+75=

76
Beast Academy Practice 2A
In a Sum Pyramid,
the number in each ds
/
block is the sum of the -

PY
rar

Beast Academy Practice 2A


To double 95,we add

To double a
number, we add
it fo itself.

= 10less than 100

PRACTICE |Solve each problem below.

93. 34+34= 94. 38+38= 95. +70+7G=

96. 234+234= 97. -882+382— 98. 390+390=

99. What number can be doubled to give the same 99.


result as 18+ 13+13413?

100. What number can be doubled to give the same 100.


result as 248+252?

78
Beast Academy Practice 2A
Double each number to get the next
PRACTICE
number in the rows below.

gam Ss [2 [Pee LT) ee


102.
ea |e eae eee
103.
Pe feos] (i) teleost ie sea
104.
Pz [| LL) Ese pra na ist ie
105.
in fina efi tnd etmafotGOa

106. Ralph doubles a number, then doubles the result 106.


and gets 92. What number did Ralph start with?

107. Winnie doubles 88. Grogg adds two 2-digit numbers 107.
and gets the same sum as Winnie. What is the
smallest number that could be part of Grogg’s sum?

Beast Academy Practice 2A


EXAMPLE Find theonessagtof 75+
6 The ones digitof \
—- { a.sum canbe found
We can add by place value: 70+60+50= 1 _\_ without finding the _
54443= 12. So, 75+64+53 =180+ 12=192. ae does
‘Theoones digitol192 is a

=
“adding tens to a ‘numberdoesn'tchange its ones :
digit. :

So, m6find heeones di


and ju
ignore the tens

PRACTICE | Answer each question below.

108. Ralph adds 12+22+32+42+52+62+72+82+92. 108.


What is the ones digit of Ralph’s result?

109. Circle the two numbers below that have a sum with ones digit 4.

243 244 245 246 247 248

. What is the smallest number of 97’s that can be added to 110.


give a result with ones digit 8?

_ . Penny adds 111 copies of 111. 111.


ie What is the ones digit of her result?

Beast Academy Practice 2A


/ This table shows
the cost in cents
. of fruits at the
\. Beast Market.

PRACTICE |Use the table above to answer each question below.

112. Find the cost of each of these purchases.

2 Bananas: cents 5 Strawberries: cents

3 Mangos: cents 4 Dragonfruits: cents

113 Circle the only amount below that could be the total cost of 6 fruits.

233 cents 234 cents 235 cents 236 cents

114. Winnie buys a bag of fruit for 186 cents. No two 114.
* fruits in the bag are the same. How many fruits are
in Winnie’s bag?

Alex buys a bag of fruit for 108 cents. All of the fruits 115.
* in Alex’s bag are the same. What fruits did Alex buy?

116. What is the largest number of fruits that can be 116.


bought for exactly 127 cents?

Beast Academy Practice 2A


EXAMPLE |Fill theblank: 35+[_]=78.
78 has 4 more tens and 3more ones than 35.
So, to get from 35 to 78, we ane « ] We con find |
4 tens and 3 ones, which is 43. | the missing
eal is 35+[43]- 78. : : number in a

EXAMPLE
te |Fi
Fillthe blank::29+(_} 86.

sxeguwin
buioeiygives
86 fae 6 more tens than 29. But, adding 6 tens to 29
89. Since this is tooates,we try 5 tens. :
Adding 5 tens 10 29 gives 79.
Then, adding ifones gives 86.
So, to get from 29 to 86, we add
5 tens and 7 ones, which iisOL,
Check: 29+|57|= 86.

PRACTICE |Fill the blank in each sum below.

117. 34+] |=88 118. 16+] |=58 119. 30+| |=91

120. 24+| |=70 121. 56+| _|=99 122. 154+| |= 182

123. 314+| |=478 124. 160+{ |= 240 125. 333+| |=518

Beast Academy Practice 2A


After the first two numbers in a Sum String, each number is the sum
____ of the two numbers to its left. For example, the Sum String below
___ starts with 3 and 10. So, the third number is 3+ 10 = 13, the fourth

Sur
Str

Next is 48+70=113, followed by 70+113=183.


OOOOOOOS
Since [11]+16 = 27, the first number in this Sum String is 11.

PRACTICE |Fill in the missing numbers in each Sum String below.

Bere ee
Bee)"CG
©, 2050 :O.0.6. 2
*

CS SOOO
* @
Beast Academy Practice 2A
Nc

It’s helpful to \__ In the problems below, cir


recognize pairs of so that every number is i
numbers that combine | pairs must be in shapes that t
fomake 10’s or, f# - 5. ee
of numbers
Below, 4 pairs of 1 S : arear circled sothat
every pair has a sum of 100.

Wo.
NT)

130. Circle eight pairs of numbers so that [18[82 [30[77


every pair has a sum of 100. [48[86[a8 [61[73

131. Circle eight pairs of numbers so that (24[18[87[53 [o8.


every pair has a sum that ends in 0.

132. Circle eight pairs of numbers so that (48[86 [ea[ee[14


* every pair has a sum that ends in 0.

Beast Academy Practice 2A


Compl
-100-Triangle,
/ the numbers in
_ the three circles
on each side
add up to 100.

PRACTICE Complete each 100-Triangle below.

133. 134. 135.

136. 0) 137. Gra} 138.


*

Find more 100-Triangle puzzles at BeastAcademy.com.

Beast Academy Practice 2A


You can add
numbers in any
order!

PRACTICE |Fill the blanks to answer each question below.

1392,” S8+91--9 140. 915+27+15 141. Lebstee:

as
ck ck is
142. 39+12+38 143. 49+36+51 144. 38 +38 +32+32

|86| Guide Pages: 88-93 Beast Academy Practice 2A


PRACTICE |Fill the blanks to answer each question below.

145. 444+234+77+184+32 146. 98+20+42+39+80

[IC -L
| aa Ws ea
—S eee

147. 79+214+85+175+56 148. 119+226+4314+74

2 -}

149. What is the sum of five 16’s and five 34’s? 149.

150. What is the sum 15+17+19+214+23+25? 150.

151. What is the sum of every whole number from 1 to 19? 151.
*

Beast Academy Practice 2A Guide Pages: 88-93


In the row of digits below, circle two The numbers
SMP
3-digit numbers whose sum is 900. | you circle in these
~ problems cannot |
5455544454 overlap.
Since 900 has ones digit 0, two numbers that sum to
900 must have ones digits whose sum ends in OF
The only way to get a sum ending iinO using two
- digits from the row iis 5+5=10. |
So, both numbers we circle must have ones ligt 5.
We find 455 +445 = 900. :

eB
Do >
Circle two numbers in each row of digits that have the
PRACTICE
given sum. The numbers you circle cannot overlap.

152. Circle two 2-digit numbers whose sum is 70.

Ie Ve2ee are Ose

153. Circle two 2-digit numbers whose sum is 100.

23344556

154. Circle two 3-digit numbers whose sum is 975.

91.8 7 OTS CAs201

155. Circle two 3-digit numbers whose sum is 789.

1 2 Br ORAsS S90) 7 / oun

Beast Academy Practice 2A


Circle two numbers in each row of digits that have the
PRACTICE
given sum. The numbers you circle cannot overlap.

156. Circle two 3-digit numbers whose sum is 432.

Wes
lig aan

157. Circle two 3-digit numbers whose sum is 710.

Tee HisHS WETBP arc yee

158. Circle two 3-digit numbers whose sum is 796.

te 4500 ee aA

159. Circle two 3-digit numbers whose sum is 558.

14414111444

160. Circle two 3-digit numbers whose sum is 850.

T° 1 1. 7A AOBNS 13 SS

161. Circle two 3-digit numbers whose sum is 800.

Seon iy CG. 8-0 2 2123 3 3

Beast Academy Practice 2A


EXAMPLE
/ The numbers -
in each blob must
be in squares that } —
\ share aside. /-
Every number
must be in exactly
one blob, and the
blobs cannot cross
each other.

_ The sums in the three blobs are


14+28+7+21 =70,
21+21+28= 70, fe
42+28=70.

PRACTICE Circle blobs of two or more numbers in each grid below.


The sum of the numbers in each blob must equal the target.

162. Target: 50 163. Target: 39 164. Target: 88

Beast Academy Practice 2A


PRACTICE Circle blobs of two or more numbers in each grid below.
The sum of the numbers in each blob must equal the target.

168. Target: 120 169. Target: 600 170. Target: 101

Beast Academy Practice 2A


PRACTICE |Answer each question below.

175. Adam adds nine 79’s. What is the 175.


ones digit of the result?

176. What is 10+11+20+22+30+33+40? 176.

177. What number can you double to equal the sum 177.
* of five 222’s?

178. There are 3 ways to add two 1-digit numbers to get 178.
* a sum of 16:

9+7, 8+8, and 7+9.

How many ways are there to add two 3-digit numbers


to get a sum of 211?

Beast Academy Practice 2A


PRACTICE |Answer each question below.

179. Grogg adds three of the numbers below and gets a sum 79;
with ones digit 3. What is Grogg’s sum?

45. 56. S672 5/6..69

180. Lizzie writes a different digit in each blank below to create 180.
a sum of three 3-digit numbers. What is the smallest
possible sum of the three numbers?

181. Winnie uses the numbers 18, 21, 25, 36, 40, and 43 to 181.
make three pairs of numbers that have the same sum.
What is the sum of each pair of numbers?

A -O-O
182. Alex splits the numbers 10, 14, 21, 22, 30, and 53 into two 182.
groups that have the same sum. What is the sum of each
group of numbers?

Beast Academy Practice 2A [93|


|94| Hints for Selected Problems Beast Academy Practice 2A
Below are hints to every problem marked with a ®&.
Work on the problems for a while before looking at the hints.
The hint numbers match the problem numbers.

152. What are the different ways we can connect 146 to 66?
174. A 3-digit number is 100 or more. What does that tell us?
9. After replacing ten e's with an X, how many X's are
175. What numbers fill the following blanks?
there?
358 is__ tens and ____ ones.
22: How else can we write KX KXKKKXK?
176. What numbers fill the following blanks?
46. Where can the 7 be placed?
77 ones is___ tens and_____ ones.
47. What is the ones digit: 5, 7, or 9?
177. What is the hundreds digit of the number?
55. What if the tens digits are all 1's? 2's? 3's? 4's?
178. How many times will 5 appear in the ones place? The
56. Why can't we circle 773?
tens place?
73. 3 hundreds is how many tens?
74. What do we get when we take away 12 ones from 222?
99. Adding 1 hundred to 792 gives 892. We must still add as There are six different 3-digit numbers that can be written
1 ten. What numbers fill the following blanks? using 2, 4, and 6 once each. Can you write them all?
892 is___tensand____ ones. 21. The number halfway between 32 and 76 is the same as
118. What numbers fill the following blanks? the number halfway between 32+ 10 and 76-10.

424 is___ tens and _____ ones. 22. The number halfway between 534 and 584 is the same
as the number halfway between 534+10 and 584-10.
122. What is Alex's number?
ZO: The number halfway between 43 and 89 is the same as
133. What two numbers can connect 182 to 83?
the number halfway between 43+10 and 89-10.
26. What number is halfway between 40 and 90?
27. What number is halfway between 350 and 410?
29. Before Player X's final move, Player O has 5 points and
Which one must we use? Player X has 3 points.
134. What two numbers can connect 91 to 100?
fe) p ame) Of OeagiAy O
ax O ©
What tick mark can Player X label to take as many points
Which one must we use? away from Player O as possible?

150. What two numbers go in the shaded squares below? 30. Before Player X's final move, Player O has 6 points and
Player X has 2 points.

ORORO xGO xX @) @)

Oyo Ox O
What tick mark can Player X label to take as many points
Which number goes in which square? away from Player O as possible?
151. What number goes in the shaded square below? 31. Before Player X's final move, Player O has 7 points and
Player X has 4 points.
00 xX X xaex

OX X O O

Beast Academy Practice 2A Hints for Selected Problems


32. Before Player X's final move, Player O has 10 points and 106. Work backwards! What number do we double to get 92?
Player X has 5 points. 107. What is the /argest number that could be part of Grogg's
X (@MO MONG (OM eN6) KOS EXPO ® OO) sum?
111. Can you ignore any of the digits in 111?
XO O Xe
114. If Winnie bought 1 fruit, what would be the ones digit of
the cost? 2 fruits? 3 fruits?
43. When completed, 3 will not be the smallest number in
this puzzle! 145. What is the ones digit of the cost of 1 fruit? 2 fruits?
3 fruits? Do you notice a pattern? What happens when
44. How can we connect 401 to 405 without blocking all of
there are 11 fruits?
the paths from 398 to 401?
45. How can we connect 15 to 19 without blocking all of the 116. What is the ones digit of the cost of 1 fruit? 2 fruits?
3 fruits? Do you notice a pattern? What happens when
paths from 19 to 21?
there are 11 fruits?
46. What are the different ways we can connect 901 to 905?
128. Start by finding the number you add to 40 to get 105.
48. Where must 103 go? Then, how many empty hexagons
are needed to connect 104 to 113? 129. Start by finding the number you add to 201 to get 524.
49. Where must 67 go? Then, how many empty hexagons 132. What number must be paired with the 46 in the top-left?
are needed to connect 68 to 78? 138. Guess what number belongs in the top circle. Did your
guess work? If not, can you make a better guess? Keep
50. How can we connect 49 to 53?
trying!
64. What digit must be placed in the blank in 46>4]?
151. What numbers pair nicely to make this sum easier?
85. Be organized! What numbers can you write with
hundreds digit 1? Hundreds digit 2? Hundreds digit 3? ia What numbers must be in the same blob as 29?
86. There is 1 missing two-digit number, and 3 missing 172. What numbers must be in the same blob as 800?
three-digit numbers. 173. All of the numbers are equal to or close to 100. We know
87. There are 3 missing two-digit numbers, and 1 missing 100+ 100+100+100=400. How does this help?
three-digit number. 174. The ones digit of the sum of every blob is 9. What
95. What numbers must go in the tens digit blanks? numbers must be in the same blob as the 13 in the
bottom-right corner?
96. What is the smallest number we can make with the given
digits? The largest number? 5
WA How could you split five 222's into two equal groups?
97. What is a good number to count in by? Think big! 178. What is the smallest three-digit number that could be in
the sum? What is the largest?
98. How should we label the tick marks between 15 and 60?
179. Do we need to consider the tens digits of any of the
99, Draw a number line!
numbers?
100. Draw a number line!
180. What digits should we use to fill the blanks in the
101. What does AHI@< tell us about the A? hundreds places?
102. Why can't Alba use 1? Why can't Alba use 9? 181. Why can't we pair 18 with 21? What number must be
103. What could be the hundreds digit of Tia's number? paired with 18?

104. What is the smallest number that could be under [a |? 182. Can you find the sum of each group without splitting the
numbers into two equal groups?
What is the largest number that could be under |m |?
105. What are the tens digits of the three numbers?
106. If you're stuck, try something! Even if it's wrong, it may
help you make a better guess.
WS

uo
37. Wha t square s can you fill in first?
38. What number goes in the middle-left square?
90. What number do we add to 23 to get 99?
9T. What number do we add to 195 to get 400?
92. Take a guess for what number belongs in the empty
bottom-middle block. Did your guess work? If not, can
you make a better guess? Keep trying!

Hints for Selected Problems Beast Academy Practice 2A


Beast Academy Practice 2A Hints for Selected Problems
MONTAG

[98| Place Value Chapter 1 Solut lions Beast Academy Pract Ice 2A
13. There are six X’s and twelve e’s in all.
We replace ten e’s with one X.

1. We draw an X for each stack of ten coins, and ae for rn bh ht aed


each extra coin. There are XK Keeeee coins.
=KXKKKKKeo
2. We draw an X for each stack of ten coins, and ae for 14. There are ten X’s and four e’s in all.
each extra coin. There are KX K XKKeeeeceecee coins. We replace ten X’s with one C.

3. We draw aC for each group of one hundred coins, an *« SKEKEKKKK Koo ove
for each stack of ten coins, and a e for each extra coin.
There are U XXX Xeee coins. =CQecce

4. We replace ten e’s with one X. 15. There are one © and eleven X’s in all.
We replace ten X’s with one ©.
mesesveasssace
CRCKKXKKKKKKX)X
=X Keee

5. We replace ten X’s with one C. =CUCUX

SEKKKKKKKM) KO
16. There are nine X’s and ten e’s in all.
We replace ten e’s with one X.
= QXe
KREKAK KEK Geerenrere
6. We replace ten X’s with one C.
=KXXKKKKKKK
SKEKEKKEKKKIOKKK Then, we replace ten X’s with one CG.
SKKKKKKKK
=OUXXX
7. We replace ten e’s with one X. =€
Bre neo 17. Captain Kraken has X&XK KK+KKKK
KKK
coins. There are twelve X’s in all.

=UXKX Keeoe We replace ten X’s with one CU.


8. We replace ten X’s with one U, and ten e’s with one X. SKKKEKKKKIOK NK
Ct EXER HED Correos
=OUXX
=CCUCX ec So, Captain Kraken now has UXX coins.
9. We replace each group of ten e’s with one X. 18. Barnacle Barney has buried UXeeeeee+ K Keececee
coins. There are one &, three X’s, and thirteen e’s in all.
KEKKKKKKK
We replace ten e’s with one X.
=KXKXKKKKKKKK Se eee
Then, we replace ten X’s with one &.
=OUXXKX Keee
XKKKKKKKMOK So, Barnacle Barney has buried UX XX Xeee coins.

=tx 19. Captain Kraken starts with & X Xeee coins, and then
takes away X Xe coins. Taking away two X‘s and one ¢
10. There are one X and eight e’s in all. from XX Keee leaves Kee.
Xeoesevee So, Captain Kraken has Xee coins left.
11. There are eight X’s and six e’s in all. 20. We split the X<‘s and the e’s into two equal groups.
Splitting * XXX coins gives two groups of KX coins.
KXKKXKKK Keoccce Splitting ¢¢eeee coins gives two groups of ¢¢@ coins.
12. There are five X’s and seven e’s in all.
So, we can make two equal piles of * Xeee coins.
XXX
X Keecccecs

Beast Academy Practice 2A Place Value Chapter 1 Solutions |99 |


21. There are three bags, with X Keeee coins in each bag. 29. We count the X’s by tens.
So, there are KXKeeee +X Keooee + XKKeeee coins all a \
iO. 20
together. x xX
This gives six X’s and twelve e’s. Then, we have 3 more e’s. 3 more than 20 is 23.
We replace ten e’s with one X.
So, X Keee=23.
AXKKK Kesooesesdee 30. We count the X’s by tens.
=KKEKKKK
Kee
So, there are KX XK XXXKee coins in the treasure
chest.
So, KX KKK =50.
22. To take away X Xeeee coins from XK XXXX coins,
. We have 1 hundred, 4 tens, and 3 ones.
we start by writing one of the &’s in KX KXKXXKX as
So, UX XK Keee= 143.
ten e’s,
. We have 1 hundred, 1 ten, and 8 ones.
KEKKKK = KKKK Kovccccccce So, UGKeeceeece=118.
Taking away two X‘s and four e’s from
. We have 3 hundreds, 2 tens, and 1 one.
XXX
XK Keccccccece leaves KX K Keeccee,
So, UCLX
Ke = 321.
So, there are KX X Keeeeee coins left in the chest.
. We have 1 hundred, 2 tens, and 4 ones.
So, UX Keeee=124.
. We have 5 hundreds, 1 ten, and 2 ones.
23. The X stands for 10 e’s, and 3 more e’s makes 13 @’s. So, CUCCOXee = 512.
So, Keee =13. . We have 1 hundred, 0 tens, and 5 ones.
24. The X stands for 10 e’s, and 8 more e’s makes 18 e’s. So, Ceeeee=105.
So, Keececceceee= 18. . We have 1 hundred, 3 tens, and 0 ones.
25. Since each XX stands for 10 e’s, we count the X’s by So, UX XX = 130.
tens. . We have 7 hundreds, 0 tens, and 0 ones.
LRN So, CLCCCCC = 700.
10 20
x xX . Only 55, 25, and 52 have two digits.
Then, we have 4 more e’s. 4 more than 20 is 24.
So, K Keecee=24,
40. Only 209 and 290 have 2 in the hundreds place.
26. We count the %’s by tens.
902.4 Yoel 929 920
LEN NEM aN,
10 20 30 40 50 60
41. Only 477, 774, and 474 have tens digit 7.
x KKK KX

Then, we have 1 more e. 1 more than 60 is 61.

So, KX KKK Ke=61. 42. Only 978, 897, and 987 have a hundreds digit larger than
27. We count the X’s by tens. their ones digit.

lan EY am 798 879 789


10 20 30 40 50
x KKK 43. We write 3 in the tens place, giving _3_. The remaining
Then, we have 4 more e’s. 4 more than 50 is 54.
digits are 2 and 2, so we write them in the remaining
place values. This gives 232.
So, XX KK Keeee= 54.
44. We write the 7 in the ones place, giving __7. The two
28. We count the X’s by tens.
remaining digits are 5 and 0. We cannot write 0 as the
CON ING leftmost digit of a three-digit number. So, we write 5 in the
10 20 30 40
x KK hundreds place and 0 in the tens place. This gives 507.

Then, we have 5 more e’s. 5 more than 40 is 45. 45. No two-digit number can have tens digit 0. So, 0 must go
in the ones place, giving _0.
So, KXKX Keeeee=45.
We can write any digit other than 0 in the tens place
(1, 2, 3, 4, 5, 6, 7, 8, or 9). So, there are 9 different
two-digit numbers that have 0 as a digit:
10, 20, 30, 40, 50, 60, 70, 80, and 90.

Place Value Chapter 1 Solutions Beast Academy Practice 2A |


46. 7 can be the hundreds digit, tens digit, or ones digit. The 56. The 3-digit numbers in this row are 733, 337, 373,
other digits are both 8’s. So, the three different three-digit 377, and 773. There is only way to circle four of these
numbers that use the digits 7, 8, and 8 are 788, 878, and numbers without overlapping.
887.
47. The ones digit is larger than the tens digit. So, the ones
digit cannot be the smallest digit, 5.
The ones digit is smaller than the hundreds digit. So, the
ones digit cannot be the largest digit, 9. 57. There are 2 hundreds, 1 ten, and 5 ones. So, there are
)
So, the ones digit is 7. Since the ones digit is larger than 215 blocks.
the tens digit, the tens digit is 5. Since the ones digit is 58. There are 2 tens, 9 ones, and 1 hundred. Ordering the
smaller than the hundreds digit, the hundreds digit is 9. place values from largest to smallest, we have
So, the number is 957. 1 hundred, 2 tens, and 9 ones. So, there are 129 blocks.
59. There are 18 blocks.

60. There are 11 tens. We combine 10 tens to make


48. There is only one 3-digit number with ones digit 2 and 1 hundred. This gives 1 hundred, 1 ten, and 0 ones.
hundreds digit 4. So, there are 110 blocks.

2342432) 61. There are 5 tens and 15 ones. We combine 10 ones to


make 1 ten. This gives 6 tens and 5 ones. So, there are
49. The different 3-digit numbers in this row are 554, 545, 65 blocks.
and 455. Only 554 and 455 have tens digit 5. There
62. There are 10 tens and 4 ones. We combine 10 tens to
is only one way to circle these two numbers without
make 1 hundred. This gives 1 hundred, 0 tens, and
overlapping.
4 ones. So, there are 104 blocks.
55 455\(455
63. There are 2 hundreds, 9 tens, and 29 ones. We combine
50. The different 3-digit numbers in this row with ones digit 20 ones to make 2 tens. This gives 2 hundreds,
9 are 999, 989, and 899. There is only one way to circle 11 tens, and 9 ones.
two of these numbers without overlapping. Then, we combine 10 tens to make 1 hundred. This

@99@999 gives 3 hundreds, 1 ten, and 9 ones.


So, there are 319 blocks.
51 The only 3-digit numbers in this row with ones digit 0
are 110 and 100. There is only one way to circle these
numbers without overlapping.

001000019 64. Every place value must contain a single digit. We cannot
OZ. The only 3-digit numbers in this row with ones digit 0 are write a number with 17 in the ones place. So, we need to
120, 200, and 220. There is only one way to circle these regroup. We can regroup 10 ones to make 1 ten.
three numbers without overlapping.

53. 314 is the only 3-digit number that appears three times in
the row.
1231421814128
142 So, 7 tens and 17 ones is the same as 8 tens and
7 ones, which is 87.
54. The only 3-digit numbers in this row whose tens digit is
the largest digit are 576, 465, and 354. 65. We cannot write a number with 28 in the ones place.
So, we regroup 20 ones to make 2 tens.

55. We look at each possible tens digit.


If the tens digit of each number is 1, we can only circle
three numbers without overlapping.
If the tens digit of each number is 3, we can only circle
two numbers without overlapping. So, 3 tens and 28 ones is the same as 5 tens and
8 ones, which is 58.
The same is true if the tens digit is 4.
If the tens digit is 2, we can circle four numbers without
overlapping.

Beast Academy Practice 2A Place Value Chapter 1 Solutions


66. We cannot write a number with 11 in the ones place. 73. We have 3 hundreds and 7 tens. To leave 0 hundreds,
So, we regroup 10 ones to make 1 ten. we break each of the hundreds into 10 tens. This gives
10+10+10=30 more tens. So, 3 hundreds and 7 tens is
the same as 30+7 = 37 tens.
74. We have 2 hundreds, 2 tens, and 2 ones.
To get 12 ones, we break 1 ten into 10 ones.
This gives 2 hundreds, 1 ten, and 12 ones.
Then, to get 0 hundreds, we break each of the hundreds
5 hundreds, 2 tens, and 11 ones is the same as into 10 tens. This gives 10+10=20 more tens for a total
5 hundreds, 3 tens, and 1 one, which is 531. of 20+1 =21 tens.

67. We cannot write a number with 15 in the tens place. So, 222 is the same as 21 tens and 12 ones.
So, we regroup 10 tens to make 1 hundred. 75. We have 6 tens and 2 ones, which is 62.
To leave 5 tens, we break 1 of the 6 tens.
Breaking 1 ten makes 10 ones.

15 tens and O ones is the same as 1 hundred, 5 tens,


and 0 ones, which is 150.

68. We have 5 tens and 26 ones. To get 6 tens, we need So, 62 is 5 tens and 10+2 12 ones.
1 more ten. We regroup 10 ones to make 1 ten: To leave 4 tens, we break 2 of the 6 tens.
Breaking 2 tens makes 20 ones.

5 tens and 26 ones is the same as 6 tens and 16 ones.

69. We have 5 tens and 26 ones. To get 7 tens, we need So, 62 is 4 tens and 20+2 = 22 ones.
2 more tens. We regroup 20 ones to make 2 tens:
To leave 3 tens, we break 3 of the 6 tens.
Breaking 3 tens makes 30 ones.

5 tens and 26 ones is the same as 7 tens and 6 ones.

70. We have 7 tens and 14 ones. To get 8 tens, we need So, 62 is 3 tens and 30+2 =32 ones.
1 more ten. We regroup 10 ones to make 1 ten:
We fill the chart as shown.

7 tens and 14 ones is the same as 8 tens and 4 ones.

71. We have 7 tens and 14 ones. To get 6 tens, we need 1


less ten. We break 1 ten into 10 ones:
76. We have 4 tens and 7 ones, which is 47.
To leave 3 tens, we break 1 of the 4 tens.
Breaking 1 ten makes 10 ones.

7 tens and 14 ones is the same as 6 tens and 24 ones.


72. We have 12 tens. To leave 2 tens, we regroup 10 tens to
make 1 hundred. So, 12 tens is the same as 1 hundred
and 2 tens. So, 47 is 3 tens and 10+7 =17 ones.

Place Value Chapter 1 Solutions Beast Academy Practice 2A


To get 27 ones, we break 2 tens to make 20 ones. 81. 91 is:

° 7 tens and 21 ones


e 1 ten and 81 ones
e 2tens and 71 ones
e 9tens and 1 one

So, 47 is 2 tens and 20+7 =27 ones. ak ono wn


To leave 1 ten, we break 3 of the tens.
Breaking 3 tens makes 30 ones. ! 82. 77s:
e 6 tens and 17 ones
e 1 ten and 67 ones
e 7 tens and 7 ones
¢ Otens and 77 ones
We fill the chart as shown.
So, 47 is 1 ten and 30+7 =37 ones.
Waerfill'the chart as shown. 83. 890 is 8 hundreds and 9 tens.
We can break 8 hundreds to make 80 tens.
So, 890 is 80+9 =89 tens.
605 is 6 hundreds and 5 ones.
We can break 6 hundreds to make 60 tens.
So, 605 is 60 tens and 5 ones.
534 is the same as 5 hundreds and 34 ones.
We can break 5 hundreds to make 50 tens.
So, 534 is 50 tens and 34 ones.

84. We can regroup 10 tens to make 1 hundred.


77. 39 is: So, 17 tens is 1 hundred and 7 tens, which is 170.
e Otens and 39 ones We can regroup 90 tens to make 9 hundreds.
e 1 ten and 29 ones So, 90 tens is 900.
e 2tens and 19 ones
We can regroup 40 of the 44 tens to make 4 hundreds.
e 3 tens and 9 ones
That gives 4+4 =8 hundreds and 4 tens, which is 840.
We fill the chart as shown.

2
78. 42is: z its ones di git by 1.
a a Ties

85. Adding 1 to 134 increases


e 4tens and 2 ones So, 134+1=135.
e 3tens and 12 ones
86. Adding 10 to 134 increases its tens digit by 1.
e 2tens and 22 ones
So, 134+10= 144.
e 1 ten and 32 ones
87. Adding 100 to 134 increases its hundreds digit by 1.
We fill the chart as shown. So, 134+ 100 = 234.
88. Adding 1 to 555 increases its ones digit by 1.
79. 58 is: So, 555+1 = 556.

e 1 ten and 48 ones 89. Adding 10 to 555 increases its tens digit by 1.
e 4tens and 18 ones So, 555+10=565.
e 3 tens and 28 ones 90. Adding 100 to 555 increases its hundreds digit by 1.
e 5 tens and 8 ones So, 555+100 = 655.
We fill the chart as shown. 91. Adding 1 to 768 increases its ones digit by 1.
So, 768+ 1 = 769.

80. 50is: 92. Adding 10 to 768 increases its tens digit by 1.


So, 768+ 10=778.
e 5 tens and 0 ones
e 4tens and 10 ones 93. Adding 100 to 768 increases its hundreds digit by 1.
e 3 tens and 20 ones So, 768+ 100 = 868.
e 1 ten and 40 ones 94. Since 11 is 1 ten and 1 one, adding 11 to 987 increases
We fill the chart as shown. its tens digit by 1 and its ones digit by 1.
So, 987+11=998.

Eocsincademysfractice 2A Place Value Chapter 1 Solutions


95. Since 101 is 1 hundred and 1 one, adding 101 to 798 114. We cannot subtract 10 from 201 by decreasing its tens
increases its hundreds digit by 1 and its ones digit by 1. digit by 1. But, since 200 is the same as 20 tens, 201 is
So, 798+101 =899. 20 tens and 1 one. Taking away 1 ten leaves 19 tens and
1 one, which is 191.
96. Since 110 is 1 hundred and 1 ten, adding 110 to 879
increases its hundreds digit by 1 and its tens digit by 1. So, 201-10=191.
So, 879+110 = 989. 115. For the first four steps, we increase the tens digit by 1.
97. 11 is 1 ten and 1 one. So, adding 11 to 729 increases its +10 +10 +10 +10 +10 +10 +10 +10

tens digit by 1, and adds 1 more. 130, 140, 150, 160, 170, 180, 190, __
Increasing the tens digit of 729 by 1 gives 739, and 1
We cannot increase the tens digit of 190. Since 190 is
more is 740. So, 729+11=740.
19 tens, adding 1 ten to 190 gives 20 tens, which is 200.
98. 101 is 1 hundred and 1 one. So, adding 101 to 279 Then, we increase the tens digit by 1 again to get 210.
increases its hundreds digit by 1, and adds 1 more. +10 +10 +10 +10 +10 +10 +10 +10
a
Increasing the hundreds digit of 279 by 1 gives 379, and 170, 180, 190, 200, 210.
130, 140, 150, 160,
1 more is 380. So, 279+101 =380.
116. For the first two steps, we increase the tens digit by 1.
hey To add 110 to a number, we add 1 hundred and 1 ten.
+10 +10 +10 +10 +10 +10 +10 +10
Adding 1 hundred to 792 gives 892. We cannot add 1 ten fo. Seer
to 892 by increasing its tens digit by 1. 255, 265, _2/5ye2Zeon e200, =,
But, since 890 is the same as 89 tens, 892 is 89 tens and Then, since 290 is 29 tens, 295 is 29 tens and 5 ones.
2 ones. Adding 1 ten gives 90 tens and 2 ones, which is Adding 1 ten gives 30 tens and 5 ones, which is 305.
902.
For the last three steps, we increase the tens digit by 1.
So, 792+110=902. +10 +10 +10 +10 +10 +10 +10 +10
A a
100. Subtracting 1 from 615 decreases its ones digit by 1. 255, 265, 275, 285, 295, 305, 315, 325, 335.
So, 615-1 =614.
We Since 580 is 58 tens, 582 is 58 tens and 2 ones.
101. Subtracting 10 from 615 decreases its tens digit by 1.
So, 615-10 =605. Adding 1 ten gives 59 tens and 2 ones, which is 592.
Adding 1 more ten gives 60 tens and 2 ones, or 602.
102. Subtracting 100 from 615 decreases its hundreds digit by 1.
So, 615-100 =515. From 602, we continue increasing the tens digit to get
612 and 622.
103. Subtracting 1 from 888 decreases its ones digit by 1.
FIO at 10" 10" 10T* E1Or™ Or rl wetaG
So, 888-1 = 887. ro ee “an ok
104. Subtracting 10 from 888 decreases its tens digit by 1. —_) — 562, 572, 582, 592, G02, Giz, G2z:
So, 888-10 = 878. To the /eft of 562, we decrease the number of tens by 1
105. Subtracting 100 from 888 decreases its hundreds digit by 1. to fill in 552 and 542 as shown.
So, 888-100 = 788. +10 +10 +10 +10 +10 +10 +10 +10
ow MeAZO
106. Subtracting 1 from 432 decreases its ones digit by 1. 542, 552, 562, 572, 582, 592, 602, 612, 622.
So, 432-1 = 431.
118. Since 420 is 42 tens, 424 is 42 tens and 4 ones.
107. Subtracting 10 from 432 decreases its tens digit by 1.
Adding 1 ten gives 43 tens and 4 ones, which is 434.
So, 432-10 = 422.
Adding 1 more ten gives 44 tens and 4 ones, or 444.
108. Subtracting 100 from 432 decreases its hundreds digit by 1. +10 +10 +10 +10 +10 +10 +10 +10
So, 432-100 = 332. yo”
109. 424, 434, 444.
11 is 1 ten and 1 one. So, subtracting 11 from 543 ———— ’ ’ ’ — ———'

decreases its tens digit by 1 and its ones digit by 1. To the left of 424, we decrease the number of tens by 1.
So, 543-11 = 532. Subtracting 1 ten gives 41 tens and 4 ones, which is 414.
110. 101 is 1 hundred and 1 one. So, subtracting 101 from Subtracting 1 more ten gives 40 tens and 4 ones, or 404.
354 decreases its hundreds digit by 1 and its ones digit Subtracting 1 more ten gives 39 tens and 4 ones, or 394.
by 1. So, 354-101 = 253. We continue the pattern to the left by decreasing the
111. 110 is 1 hundred and 1 ten. So, subtracting 110 from 435 number of tens by 1 to fill in 384, 374, and 364 as shown.
decreases its hundreds digit by 1 and its tens digit by 1. othe) +10 +10 +10 +10 +10 +10 +10
ra wn’
So, 435-110 = 325. 364, 374, 384, 394, 404, 414, 424, 434, 444.
112. Counting down from 120, we get 120-1=119. 119. 800 is 8 hundreds, 0 tens, and 0 ones.
113. Subtracting 100 from 102 decreases its hundreds digit by 1. Breaking a hundred into 10 tens gives 7 hundreds,
This leaves 0 hundreds, 0 tens, and 2 ones, which is 2. 10 tens, and 0 ones.
So, 102-100=2. Taking away 1 ten leaves 7 hundreds, 9 tens, and 0 ones,
which is 790.

Place Value Chapter 1 Solutions Beast Academy Practice 2A |


703 is 7 hundreds, 0 tens, and 3 ones.

[e[m[oe] sar[saa
Breaking a hundred into 10 tens gives 6 hundreds, YOO OE,
10 tens, and 3 ones.
Taking away 1 ten leaves 6 hundreds, 9 tens, and 3 ones,
which is 693.
309 is 3 hundreds, 0 tens, and 9 ones. =10 =10 +10 +10

sofas] to.[see
Breaking a hundred into 10 tens gives 2 hundreds, a LS a= ae
10 tens, and 9 ones.
128.
Taking away 1 ten leaves 2 hundreds, 9 tens) and 9 ones,
which is 299.

=i Al -10 -10

ror]100]se] 191.[aoe]
sea]se
8 hundreds is 80 tens. Taking away 1 ten leaves 79 tens, CE fa
which is 790.
7 hundreds is the same as 70 tens. So, 703 is 70 tens
and 3 ones. Taking away 1 ten leaves 69 tens and
3 ones, which is 693. = OOM 100
Ya fae
3 hundreds is the same as 30 tens. So, 309 is 30 tens
and 9 ones. Taking away 1 ten leaves 29 tens and
9 ones, which is 299.
120. We cannot subtract 10 from 506 by decreasing its tens
. TO get from 62 to 82, we add 10 twice. So, the number in
digit by 1. But, 5 hundreds is the same as 50 tens. So,
the square between 62 and 82 is 62+10=72.
506 is 50 tens and 6 ones. Taking away 1 ten leaves
+10 +10
49 tens and 6 ones, which is 496.
So, there are 496 chipskunks left in the field.
121. We cannot subtract 10 from 405 by decreasing its tens
digit by 1. But, 4 hundreds is the same as 40 tens. So, To get from 182 to 83, we subtract 100 and add 1. So,
405 is 40 tens and 5 ones. Taking away 1 ten leaves the number in the square between 182 and 83 is either
39 tens and 5 ones, which is 395. 182-100 = 82 or 182+ 1 = 183.
So, Captain Kraken has 395 lollipops left in his treasure. Since 82 already appears in the grid, we write 183
122. We work backwards. Winnie’s number is 209. Alex’s between 182 and 83.

e[»l=[=
+1 -100
number is 100 less than Winnie’s number, so Alex’s ak.
number is 209-100 = 109.
Lizzie’s number is 10 less than Alex’s number.
So, Lizzie’s number is 109-10.
134. To get from 91 to 100, we add 10 and subtract 1. So,
1 hundred and 9 ones is the same as 10 tens and 9 ones. the number in the square between 91 and 100 is either
Taking away 1 ten leaves 9 tens and 9 ones, which is 99. 91+10=101 or 91-1=90.
So, Lizzie’s number is 99.
Since 90 already appears in the grid, we write 101 in the
Grogg’s number is 1 less than Lizzie’s number. So, empty square between 91 and 100.
Grogg’s number is 99-1 =98. +10 -1

soo]
eo] | |
To get from 90 to 99, we add 10 and subtract 1. So,
123. To get from 63 to 65, we add 1 ie EZ
the number in the square between 90 and 99 is either
twice. So, the number in the empty 63 |64|65.
90+10=100 or 90-1 =89.
square is 63+1 = 64.
Since 100 already appears in the grid, we write 89 in the
empty square.

]e|ss] 1[i]t]
+100 +100 +10 +10 =

ao |
+10
SATS f«Z™ Vi
124.

Beast Academy Practice 2A Place Value Chapter 1 Solutions


135. To get from 17 to 28, we add 10 i 143.

=D
and add 1. So, the number in the 17 |18
|
empty square is either 17+ 1=18 0 v00(
or 17+10=27. Since 27 already
appears in the grid, we write 18
in the empty square.
144.

-10(

SS /

145. Final:

( te
138. 139. |179
De

SE 3\2
10

140.

141. To get from 9 to 20, we add 1 and add 10. So, the
number in the bottom-left square is either 9+ 1=10 or
9+10=19. Since 10 already appears in the grid, we write
19 in the bottom-left square.

To get from 10 to 120, we add 10 and add 100. So, the


number in the top-right square is either 10+10 =20 or
10+100 = 110. Since 20 already appears in the grid, we
write 110 in the top-right square.
+100

148. Step 1: Step 2:

142. =

Tt

Place Value Chapter 1 Solutions Beast Academy Practice 2A _


149. Step 1: 152. From 146 to 66, we subtract 100, add 10, and add 10.
There are three different ways we could order these:
—100, +10, +10;
+10, = 100) +10;
+10, +10, -100.

We look at a path between 146 and 66, and try all three:

150. To get from 103 to 13, we subtract 100 and add 10. So,
the two shaded squares below are 103-100 =3 and
103+10=113.

3 or 113
If we write 3 in the bottom-center square, then the
bottom-right square must be 13. But, 13 already appears
in the grid, so we cannot use this placement.

Two of the grids must use the same number twice. So,
we cross out those grids out and keep the only solution.

So, we write 3 in the top-left square and 113 in the


bottom-center square.

153. Adding 5 hundreds to 213 increases its hundreds digit by 5.


So, 213+500 =713.
154. Adding 5 ones to 213 increases its ones digit by 5.
So, 213+5=218.
Then, to get from 113 to 23, we add 10 and subtract 100.
So, the bottom-right square is 1138+10= 123 or 155. Adding 5 tens to 213 increases its tens digit by 5.
113-100 =13. Since 13 already appears in the grid, we So, 213+50 = 263.
write 123 in the bottom-right square. 156. Subtracting 4 ones from 586 decreases its ones digit by 4.
So, 586-4 = 582.
157. Subtracting 4 hundreds from 586 decreases its hundreds
digit by 4. So, 586-400 = 186.
158. Subtracting 4 tens from 586 decreases its tens digit by 4.
So, 586-40 = 546.
151. We first look at the top-center square that touches 114, 159. Adding 7 tens to 121 increases its tens digit by 7.
24, and 125. So, 121+70=191.

To get from 114 to 24, we add 10 and subtract 100. So, 160. Adding 7 ones to 121 increases its ones digit by 7.
we could write 124 or 14 in the top-center square. But, 14 So, 121+7=128.
is not 1, 10, or 100 apart from 125. Since 124+1=125, 161. Adding 7 hundreds to 121 increases its hundreds digit by 7.
we write 124 in the top-center square. So, 121+700 = 821.
Then, we fill in the remaining squares. 162. Subtracting 3 hundreds from 475 decreases its hundreds
Step1: Final: digit by 3. So, 475-300 = 175.
+10 -100
163. Subtracting 3 tens from 475 decreases its tens digit by 3.
So, 475-30 = 445.
164. Subtracting 3 ones from 475 decreases its ones digit by 3.
So, 475-3 = 472.

Beast Academy Practice 2A Place Value Chapter 1 Solutions


165. 4 tens and 4 ones plus 1 ten and 7 ones is 4+1 =5 tens 170. 446 is 4 hundreds, 4 tens, and 6 ones. We cannot take
and 4+7=11 ones. We cannot write an 11 in the ones away 8 tens from 4 tens.
place. So, we regroup 10 ones to make 1 ten. We break 1 hundred into 10 tens. This gives
3 hundreds, 14 tens, and 6 ones.

Then, subtracting 8 tens leaves 3 hundreds, 14-8 =6


tens, and 6 ones, which is 366.
So, 446-80 = 366.
This gives 6 tens and 1 one, which is 61. So,
44+17=61. 440 is 44 tens. So, 446 is 44 tens and 6 ones. Taking
166. We cannot subtract 7 ones from 4 ones. So, we break 1 away 8 tens leaves 44-8 = 36 tens and 6 ones, which is
ten into 10 ones. 366.

171. 111 is 1 hundred, 1 ten, and 1 one. So, adding 111 to


789 gives 8 hundreds, 9 tens, and 10 ones.
We regroup 10 ones to make 1 ten. This gives
So, 44 is the same as 3 tens and 14 ones.
8 hundreds, 10 tens, and 0 ones.
Subtracting 1 ten and 7 ones leaves 3-1 =2 tens and
14-7=7 ones. Then, we regroup 10 tens to make 1 hundred. This gives
9 hundreds, 0 tens, and 0 ones, which is 900.
So, 111 more than 789 is 900.
172. 0 cannot be the leftmost digit of a three-digit number.
So, the leftmost digit must be 1 or 2.

So, 44-17 =27. If 1 is the leftmost digit, the other two digits are 0 and 2.
There are two ways to order them: 102 and 120.
167. There are ten X’s and six e’s in all.
If 2 is the leftmost digit, the other two digits are 0 and 1.
KKXKKKKKXKK Kecccce There are two ways to order them: 201 and 210.
We replace ten X’s with one CU. So, the four different three-digit numbers that use the
SKKEKKKK KM)coccce digits 0, 1, and 2 are 102, 120, 201, and 210.
173. 789 is 7 hundreds, 8 tens, and 9 ones.
=Ceccece
7 hundreds means there are 7 V's in the pirate number.
So, we have 8 tens means there are 8 X’s in the pirate number.
KKK KOC+KKKKK Keoooe=Cocccee= 106. 9 ones means there are 9 e’s in the pirate number.

So, 42+64
= 106. So, there are 7+8+9 = 24 symbols in the pirate number
for 789.
168. We cannot take away four X’s from one X. So, we
break the © in UX¢e¢e into ten X’s. 174. Since we are writing a 3-digit number, the number must
be at least 100, and use at least one \. We can use any
RUKee= KKK KKKKKXKK
Keo of the three pirate symbols as the second symbol. So, we
Taking away four %’s leaves KXXXX
KX Kee=72. have Ve, UX, and Ct.
So, 112-40=72. Using digits, these numbers are 101, 110, and 200.
169. 683 is 6 hundreds, 8 tens, and 3 ones. Adding 7 tens to 175. 358 is 35 tens and 8 ones. To fill the missing blanks, we
683 gives 6 hundreds, 8+7 = 15 tens, and 3 ones. either count up or down by 3 tens from 35 tens. Adding 3
We regroup 10 tens into 1 hundred. This gives tens to a number does not change its ones digit.
7 hundreds, 5 tens, and 3 ones, or 753. +30 +30 +30 +30 +80 +80 +30 +30
pie
So, 683+70 = 753. 178, 208, 238, 268, 298, 328, 358, 388, 418.
= Of —
176. 77 tens plus 77 ones is the same as 77 tens plus 7 tens
680 is 68 tens. So, 683 is 68 tens and 3 ones. Adding 7 and 7 ones. This gives 77+7 = 84 tens and 7 ones.
tens gives 68+7 =75 tens and 3 ones, which is 753. 84 tens is 840, so 84 tens and 7 ones is 847.
177. To make the smallest three-digit number possible, we
Start by making the hundreds digit as small as possible.
The hundreds digit of a three-digit number cannot be 0.
So, we consider numbers with hundreds digit 1.
1

Place Value Chapter 1 Solutions Beast Academy Practice 2A |


Then, we make the tens digit as small as possible. Since
the sum of all three digits is 12, the tens and ones digits
must have a sum of 12-1 =11.
If the tens digit is 0, then the ones digit must be 11. But,
we cannot write 11 in the ones place.
If the tens digit is 1, then the ones digit must be 10. But,
we cannot write 10 in the ones place.
If the tens digit is 2, then the ones digit must be 9. This
works!
So, 129 is the smallest three-digit number with a digit
sum of 12.
178. When writing numbers between 1 and 100, the digit 5
can only appear in the ones or the tens place.
The numbers less than 100 with ones digit 5 are:
Sheek ks Hira) pceoe hashed olaped fo eaoto a)
So, the digit 5 appears ten times in the ones place.
The numbers less than 100 with tens digit 5 are:
DOWO ly oe, oo, 0400,.00, 07, oo) OG.
So, the digit 5 appears ten times in the tens place.
So, when writing every number from 1 to 100, you will
write the digit 5 a total of 10+10=20 times.
Note that there are only 19 numbers that include the
digit 5, but since 55 has two fives, we write the digit 5
20 times.

Beast Academy Practice 2A Place Value Chapter 1 Solutions


}
|
|
Place Value Chapter 1 Solutions Beast Academy Practice 2A _
+50 +50 +50

een eee
150 300 eal cae! asl
We count by 50’s to fill in the boxes as shown below.


eet
150 300

1 Before filling the boxes, it is helpful to label the big tick


marks. The tick marks on this number line count by 50’s,
so the big tick marks are 50+50 = 100 units apart.

100 200 Fs a e i 700

There are six 3-digit numbers we can make using the


digits 2, 4, and 6 once each:
246, 264, 426, 462, 624, and 642.
The only number in our list between 250 and 300 is 264.
16 20 25 43 The only number in our list between 400 and 450 is 426.
The only number in our list between 450 and 500 is 462.
Both 624 and 642 are between 600 and 650.
Since the arrow points much closer to 650 than to 600,
we label the rightmost arrow with 642.
We label these four numbers as shown below.

34 45 ol 56. 62: (67


100 200 300 male ae
= 1426] |462 642
8. We move two tick marks to get from 80 to 100. YUE

So, the tick marks on this number line count by 10’s.


MK We
+10 +10
12. From 19 to 20 is 1 unit.
<—_—_——————-——-+——-—+
+ ooo >
From 20 to 60 is 10+10+10+10=40 units.
eel 80 100 eal 150 ad
+1 +10 +10 +10 +10

We count by 10’s to fill in the boxes as shown below.


420 60
aa a eee eee ee ee ee ei Sal
19
80 100 150
So, the distance from 19 to 60 is 1+40 = 41 units.
9. We move six tick marks to get from 320 to 380. The left dot is on the number 84, and the right dot is on
So, the tick marks on this number line count by 10’s. the number 124.
+10 +10 +10 +10 +10 +10
From 84 to 90 is 6 units.
a ee From 90 to 120 is 10+10+10=30 units.
[a] 320 [nal 380 [3] From 120 to 124 is 4 units.
+6 +10 +10 +10 +4
We count by 10’s to fill in the boxes as shown below. Yo ed Nt NS eae
ij
i
320 380 h 90 100 110 120
84 124
10. We move three tick marks to get from 150 to 300. So, the distance between the two dots is 6+30+4 = 40
So, the tick marks on this number line count by 50’s. units.

Beast Academy Practice 2A Comparing Chapter 2 Solutions |111 |


84 and 124 have the same ones digit. So, we can count To find the other number that is 25 units from 73, we
up by 10’s to get from 84 to 124. count down 25 units from 73.
+10 +10 +10 +10 =5. 10 =O +10 +10 +5
ee On OS ee A A A A A A
i. 90 # 100 110 120 } 48 53 63 73 83 93 9
84 94 104 114 124
So, 98 and 48 are the same distance from 73.
So, the distance between the two dots is
10+10+10+10=40 units.

14. From 54 to 60 is 6 units.


18. We count in from 45 and 65.
From 60 to 80 is 10+10=20 units.

Seen fas:
+5 =5
From 80 to 81 is 1 unit.
ae EAL AAAS CLLELEAE SARA A A
+6 +10 i OMe
Fe
SENN ee 45 50 60 65
40 50 * 60 70 80} The number halfway between 45 and 65 is the same as
54 81 the number halfway between 50 and 60.

So, the distance from 54 to 81 is 6+20+1 =27 units.

We count up by 10’s to get from 54 to 74. Then, we count


up by 7 more to get to 81. The number halfway between 50 and 60 is 55.
+10 +10 +7 So, the number halfway between 45 and 65 is 55.
ge Hee FON 19. We count in from 210 and 260.
40 i Ce SO ee,
54 64 Tae: EI ak my8| eS See

~ctifeichclehatbefeteabnbat chepapb penne tpl tahaeattrhat cbt


So, the distance from 54 to 81 is 10+10+7 = 27 units. 210 220 230 240 250 260
15. The distance from 18 to 25 is 2+5=7 units.
The number halfway between 210 and 260 is the same
as the number halfway between 230 and 240.
+10 +10 ae gee =10 -10

Aa
To find the other number that is 7 units from 25, we count 230 235 240 250 260
up 7 units from 25.
The number halfway between 230 and 240 is 235.
+2 +5 +5 +2
So, the number halfway between 210 and 260 is 235.
ROI,
ice aN a |
A A A 20. We count in from 27 and 73.
18 25 32
+3 ty

So, 18 and 32 are the same distance from 25.


/* a
57 WidMAASAI MAAADDABAL MAAR BAAS DADAAAA DAA LAPGAAAAA’ SALLE
16. The distance from 33 to 50 is 7+10=17 units. 27 30 40 50 60 70 73

The number halfway between 27 and 73 is the same as


DEN Alias reer Rrra cc ONE the number halfway between 30 and 70.
33 50 +3 +10 +10 =10 -10 -3
LF ee NL eee
To find the other number that is 17 units from 50, we
a A
count up 17 units from 50. 27 30 40 50 60 70 73

The number halfway between 30 and 70 is 50.


So, the number halfway between 27 and 73 is 50.
33 50 67 21. If we try to count in to 40 and 70, then we must count in
So, 33 and 67 are the same distance from 50.
by different amounts.
+8 -6
17. The distance from 73 to 98 is 10+10+5 =25 units. ae, | | See

t A t + t tet OAABOAE DARAAS OARS Bn


32 40 70 76 |

When finding the number halfway between two other


numbers, we must always count in by the same amount.
So, we try something else.

1112| Comparing Chapter 2 Solutions Beast Academy Practice 2A.


Since it is easy to add and subtract 10’s, we try counting The distance from 40 to 64 is 10+10+4 = 24 units.
in by 10’s from 32 and 76. The distance from 64 to 90 is 6+10+10= 26 units.
So, 64 is closer to 40 than it is to 90.

a ear A A
27. The number halfway between 350 and 410 is the same as
32 42 52 56 66 76
the number halfway between 360 and 400, which is
the number halfway between 370 and 390, which is 380.
The number halfway between 32 and 76 is the same as
the number halfway between 52 and 56. 385 is more than 380. So, 385 is closer to 410 than it is
to 350.
+10 +10. +2-2 -10 -10
Po ee NAY eee
A A A A A A
32 42 52 {56 66 76 The distance from 350 to 385 is 10+10+10+5 = 35 units.
54
The distance from 385 to 410 is 5+10+10=25 units.
The number halfway between 52 and 56 is 54.
So, 385 is closer to 410 than it is to 350.
So, the number halfway between 32 and 76 is 54.
22. We count in by 10’s from 534 and 584.

a NS ge So
28. Only by labeling the X shown below can Player X win.

A A A A A A
534 544 554 564 574 584
O X Cs
XO Wee eX
The number halfway between 534 and 584 is the same We score the game as shown below.
as the number halfway between 554 and 564.
+10 +10 Ree SU) -10

A A A A a A
@ x Oy eX ee rae
534 544 554 t 564 574 584
559 X scores 1 point
O scores 0 points
The number halfway between 554 and 564 is 559.
So, the number halfway between 534 and 584 is 559.
29. Only by labeling the X shown below can Player X win.
23. We count in by 10’s from 70 and 130.
The number halfway between 70 and 130 is the same as
the number halfway between 80 and 120, which is os OX bo
the number halfway between 90 and 110, which is 100. We score the game as shown below.
So, the number halfway between 70 and 130 is 100.
Oo xX O Mary Xb 3X e)
24. We count in by 2 from 118 and 162.
The number halfway between 118 and 162 is the same as
os O X ee
the number halfway between 120 and 160. X scores 4 points
O scores 3 points
Then, we count in by 10’s from 120 and 160.
The number halfway between 120 and 160 is the same as
the number halfway between 130 and 150, which is 140. 30. Only by labeling the X shown below can Player X win.
So, the number halfway between 118 and 162 is 140.
25. We count in by 10’s from 43 and 89. yee 4 O X O
The number halfway between 43 and 89 is the same as We score the game as shown below.
the number halfway between 53 and 79, which is
» eres ree) ye @) @)
the number halfway between 63 and 69. as RIL SEESn et A LR Rn ea
The number halfway between 63 and 69 is 66. KeG sy ox rx O
So, the number halfway between 43 and 89 is 66. X scores 4 points
Oscores 3 points
26. To find out if 64 is closer to 40 or to 90, we find the
number that is halfway between 40 and 90.
The number halfway between 40 and 90 is the same as
the number halfway between 50 and 80, which is
the number halfway between 60 and 70, which is 65.
64 is less than 65. So, 64 is closer to 40 than it is to 90.

Beast Academy Practice 2A Comparing Chapter 2 Solutions [113


31. Only by labeling the X shown below can Player X win. Then, there is only one way to place 6 and 7 to connect

Tee Lonsn | CLI


ee oes ie
We score the game as shown below. Cece |
00 Veh cE eer ite; oes |
OX X Qe O
X scores 6 points Finally, we complete the puzzle by placing 10 as shown.
O scores 5 points

32. Only by labeling the X shown below can Player X win.

XO OX pee
We score the game as shown below.

xX 0000 Eee
SO LS SO) TOMO) ;
We use the strategies from the previous problems to
xO OX ~) SEG) complete the following puzzles.
X scores 8 points
O scores 7 points

33. We start by placing 2 to connect 1 to 3.

Then, there is only one way to place 5 and 6 to connect


4 to 7.

We complete the puzzle by placing 8 to connect 7 to 9.

34. There is only one way to place 3 and 4 to connect 2 to 5.

Comparing Chapter 2 Solutions


There are two empty hexagons touching 7. One of these We use the strategies from the previous problems to
hexagons must contain an 8. If we place the 8 below complete the following puzzles.
the 7, then our path of consecutive numbers cannot
cross every hexagon.

So, we place the 8 above the 7. Then, we fill the empty


hexagons to the left of 8 with 9 and 10.

Finally, we can only fill the empty hexagons to the right of


3 with 2 and 1, as shown below.

44. 398 and 401 must be connected by 2 empty hexagons.


401 and 405 must be connected by 3 empty hexagons.
There is only one way to connect 398 and 401 with two 51 . 98 is greater than 89.
empty hexagons and connect 401 and 405 with three So, we fill the circle with the “is greater than” symbol.
empty hexagons, as shown below.
98 (>) 89
52. 2+7=9, and 9 is less than 27.
So, we fill the circle with the “is less than” symbol.

2+7 (<) 27
53. 9+3 and 3+9 both equal 12.
So, we fill the circle with the “is equal to” symbol.

9+3 (=)3+9
54. 40+4=44, and 50-5=45.
44 is less than 45. So, we fill the circle as shown.

40+4 (<)50-5
55. 213 is greater than 132. So, we fill the circle as shown.

We complete the puzzle by filling the remaining empty so ©) 192


hexagons with 397 and 396. 56. 7+80=87, and 70+8=78.
87 is greater than 78. So, we fill the circle as shown.

7+80 (>)70+8
57. 19 ones is 19, and 2 tens is 20.
19 is less than 20. So, we fill the circle as shown.

19 ones (<) 2 tens

Beast Academy Practice 2A Comparing Chapter 2 Solutions 115 |


58. 30 tens is the same as 3 hundreds, or 300. number in the 200’s. So, 323 is largest.
So, we fill the circle as shown. Bes ee ae
30 tens G) 3 hundreds Finally, since 23 is less than 33, we know 223 is less
than 233. From least to greatest, we have
59. 199 is 1 less than 200, and 202 is 2 more than 200.
So, the sum 199+202 is 1 more than 200+200. 32, 223, 233, 323.

We fill the circle as shown. . We have two numbers in the 400’s and two numbers
in the 700’s. Any number in the 400’s is smaller than
199+202 (>) 200+200 any number in the 700’s. So, 471 and 417 are the two

60. 99 is 1 less than 100. So, the sum of three 99’s is 3 less smallest numbers.
than the sum of three 100’s. Since 17 is less than 71, we know 417 is less than 471.
So, 99+99+99 and 300-3 are equal. So, the first number is 417, followed by 471.
AVG AT,
99+99+99 (=) 300-3
Since 14 is less than 41, we know that 714 is less than
61. 60 is the only two-digit number with tens digit 6 that is 741. So, the next number is 714, followed by 741.
less than 61. So, we fill the blank with 0.
From least to greatest, we have
6[0|<61 417, 471, 714, 741.
62. Filling the blank with a digit that is 8 or less gives a result . To make the largest three-digit number whose digits are
that is 84 or less, which is not greater than 85. all different, we use the three largest digits: 7, 8, and 9.
So, we can only fill the blank with 9. Any number in the 900’s is greater than any number in

[9]4>85 the 700’s or 800’s. So, we write 9 in the hundreds place.


9
63. We start by looking at 7L] >73. Among the digits 2, 3,
We must fill the remaining place values with 7 and 8.
and 4, only 4 can fill this blank.
Since 987 is greater than 978, the greatest number we
Then, we look at 30<[_]1. Among the remaining digits, can make is 987.
2 and 3, only 3 can fill this blank.
» Tomake the smallest three-digit number whose digits are
Finally, we fill the blank in []3<34 with the only all different, we use the three smallest digits: 0, 1, and 2.
remaining digit, 2.
The hundreds digit cannot be 0, otherwise we get a
30<([3]1 two-digit number, such as 012 (which is just 12).

7|4|>73 So, the hundreds digit must be 1 or 2. Any number in the


100’s is smaller than any number in the 200’s. So, we
[213 <34 write 1 in the hundreds place.
1

64. We start by looking at 46>4[]. Among the digits 5, 6, 7,


and 8, only 5 can fill this blank: 46>45]. We must fill the remaining place values with 0 and 2.
Then, we look at 7L]<]5. The number 7L] must be Since 102 is less than 120, the smallest number we can
make is 102.
7{6], 7[Z], or 7[8]. So, the number (15 cannot be 75 or
less. So, only 8 can fill the second blank: 7L]<([8]5. . Three-digit numbers are greater than two-digit numbers
Next, we look at []1>62. Among the remaining digits which are greater than one-digit numbers.
6 and 7, only 7 can fill this blank: [Z7]1 >62. So, we order the three-digit numbers first, then the
We fill the remaining blank in 7L)<[8]5 with the only digit two-digit numbers, then the one-digit number.
we have not yet used, 6. This gives 7[6] <[8]5.

[7]1>62
ay iw
s& 5S 6? Ss S 62
WKF WK
7[6]<[8]5 785 785

46> 4[5] 111:5


51
758
51/1
23 203
le pede Tet atte
7/518 51
65. 32 is the only number less than 100, so it is the smallest. 51111 23
32, 210'3 6
Next, we have two numbers in the 200’s and one number
in the 300’s. Any number in the 300’s is larger than any

Comparing Chapter 2 Solutions Beast Academy Practice 2A |


70. We order the three-digit numbers first, then the two-digit 78.
numbers, then the one-digit numbers.
2 by

Ss Ss
WK" So”
415 543
345 4.3/5
435 345
514 5 4
4 . 53
543 45
5 i)
We have 16<18<27<61<72<108.

81. We have 650>605>560>506>65>56.


71. We order the three-digit numbers first, then the two-digit . The number that replaces the gray box must be greater
numbers, then the one-digit number. than 10 and less than 20. Only the whole numbers from
ae yw 11 to 19 are greater than 10 and less than 20:
Ss % Ss
1112413, 145151 6 elifeelO ano,
SFO" SK
1/1 919 So, 9 different numbers could replace the gray box.
san a 119 83. We have 2<9<22<29<92<99<222<229,.

fimen
99
ek91
a8
85.
We have 33<34<43<44<333<334<343<344<433.
We have 123<132<213<231<312<321.

1,1\9 19 86. We have 0<5<50<55<500<505<550<555.


9 1/1 87 We have 7<8<77<78<87<88<777</778.
119 9

88. Since the ones digits are all the same, we place 4, 7, and 8
in the tens digit blanks in order from least to greatest.

[4]2<[7]2<[8]2
89. Since the largest number is 80, we cannot place 9
in either tens digit blank. So, we must place 9 in the
leftmost blank.

[gl<[_]o<[_]o<80
Then, we place the remaining digits 3 and 5 as shown.

[9]<[3]0<[5]0<80
90. Among the given digits, only 6 can be placed in the
leftmost blank.

34<3[6]<4[_|<4[|
Then, we place the remaining digits 2 and 4 as shown.

34<3[6]<4[2]<4/4]
. Since the largest number is 60, we cannot place 6 or 7 in
either tens digit blank. So, the tens digit blanks must be
filled with 4 and 5.

aiL\<411<)4<60
Then, we place the digits 6 and 7 as shown.

[4l[6}< 4[7]<|5]4<60

Beast Academy Practice 2A Comparing Chapter 2 Solutions


92. Since the middle number has tens digit 9, the rightmost
number must also have tens digit 9.
97. There are many ways to count in from 123 and 321. For _
Ll9<9Ll<[9]7 example, we could count in by 21, then by 56, then by 20 —
Among the remaining digits 6 and 8, only 6 can be as shown below. |
placed in the ones blank of the middle number.
The number halfway between 123 and 321 is the same as |

Ll9<9[6]<[9]7 the number halfway between 220 and 224, which is 222.
—56
|

So, we place 8 in the remaining blank. +21 +56 400 -20 21

[8]9<9)6]<|9]7 100 » 144


123
200 f W 244
220 224
300
321
»

93. Since the largest number is 75, we cannot place 7 in


either tens digit blank. So, we place the 7 as shown. So, the number halfway between 123 and 321 is 222.

5<(7]<[ |5 <[ ]5<75


We count in by 100 from 123 and 321.
Then, we place the remaining digits 5 and 6 as shown.
The number halfway between 123 and 321 is the same as
5<[7]<(5]5<|6]5<75 the number halfway between 223 and 221.
94. 11 is the smallest number we can make using the given +100 - 100
digits, followed by 12, then 21, then 22. So, we fill the
blanks as shown.
100 200 f i 300
[alit}<[12}<[2)(4]<(2\f2] 123 221 223 321
95. Since the smallest number is 40 and the largest number The number halfway between 223 and 221 is 222.
is 70, the digits 3, 9, and 9 cannot be placed in any of the
So, the number halfway between 123 and 321 is 222.
tens digit blanks. So, we must place 4, 5, and 5 in the
tens digit blanks as shown. 98. We move three tick marks to get from 15 to 60.

40<[@)-1<(SC1<BIL1<70 So, the tick marks on this number line count by 15’s.
+15 +15 +15
Since 59 is not greater than 59, we cannot place both 9’s
in the blanks with tens digit 5. So, we place the 3, 9, and 9
————
15 60 fe heel CI

- 40<[4I)<IS}<Ia|<70
96. We try working from the outside in. The smallest number
We move two tick marks to get from one big tick mark
to the next. So, the big tick marks are 15+15=30 units
apart.
we can make using the given digits is 55, and the largest
We count by 30’s to label the big tick marks as shown.
number we can make is 99. So, we try filling the blanks
as shown below. S$
is) Sie 60 sa Eat hack woe qe

Digits: 8, 5, 5,5,7,7,9,%8%

Islis}<_]7<LU<7L<L/
<(9Ig) Then, we fill the remaining boxes with 120+ 15 = 135 and
180+15=195.
The next-smallest number we can make is 57, and the
next-largest number we can make is 97. So, we try the
<<
piswegete) 60 1207735] 15° Tefal
following.

Digits: 4, 5,3, 5,7,7,9,9,9 99: We label Alex’s and Lizzie’s numbers on the number line.

[slis}<[5]7 <_L\<7L\<(9]7 <[9\[9) 20 25 30 40 50 55 60 70 80 90


We fill the remaining blanks as shown below.
Alex
t t
Lizzie
Digits: 5, 5, 8, 8, X, ¥% 9,9, 8

[5|[5]<|5]7 <[7][5]<7/7}<(9]7 <[9][9} The distance between Alex’s number and Lizzie’s
number is 10+10+10=30 units.
This is the only solution.
+10 +10 +10

20 25 30 40 50 5S 60 70 80 90

Comparing Chapter 2 Solutions Beast Academy Practice 2A


Lizzie’s number is halfway between Alex’s number and 101. The largest number has hundreds digit 3. Since the A is
Grogg’s number. So, the distance between Lizzie’s the hundreds digit of the smallest number, we know the
number and Grogg’s number is also 30 units. Ais 3 or less. But, since ABj®<3H@, the A cannot be 3.
+10 +10 +10 +10 +10 +10 So, the A is 2 or less.
Looking at the tens digits in 3@@%1<3AA, we know the
20 rn30 40 50 55 60 70 80 85 90
@ is less than the A.

Alex Lizzie GrOgg


Looking at the tens digits in 381@<3@M, we know the
Eis less than the ©.
So, Grogg’s number is 85.
So, we have f<@<A, and the A is 2 or less. This leaves
100. Ms. Q marks the number 50, and Alex marks the number only one possible value for each shape.
15 units to the left of Ms. Q’s number. So, Alex marks 35.
We have
A= 2, @=1, and#=0.

ABO <3H@<308<3AA
2 OSsOi liao O<322

Alex Ms. Q 102. We start by looking at 17L1<LJLI0. Since each blank is


filled with the same digit, both [J7L] and LILO have the
Grogg marks the number 25 units to the right of Alex’s same hundreds digit.
number. So, Grogg marks 60.
So, for (JL0 to be greater than (J7L, the tens digit of
+10 +10 +5
VN SEN [IDO must be more than 7. So, the digit in each blank is
Se tl einer Gmnnnammeeniene cca either 8 or 9.

i
35 50 60
Next, we look at LJL]0<9LJO. If the digit in each blank is 9,
Alex Ms. Q Grogg then LJLJ0 will be equal to 9LJ0.
Lizzie marks the number 35 units to the left of Grogg’s So, the digit Alba uses in each blank is 8.
number. So, Lizzie marks 25.
(8]7[8] <[8][8]0<9[8]o
an a 103. Since Tia’s favorite number is between 777 and 999, the

Ries foey RobeBon 3p


hundreds digit of her number is 7, 8, or 9.
25 35 50 60
The hundreds digit is less than the ones digit, which is
Lizzie Alex Ms.Q — Grogg
less than the tens digit. This means that the hundreds
Winnie marks the number 45 units to the right of Lizzie’s digit is at least two less than the tens digit.
number. So, Winnie marks 70. So, the hundreds digit cannot be 8 or 9 and must be 7.
Since the hundreds digit of Tia’s number is 7, the ones
digit is 8, and the tens digit is 9.

en en
25 35 50 60 70
So, Tia’s favorite number is 798.
Lizzie Alex Ms. Q Grogg Winnie
104. The smallest number that could be under [@] is 16.
So, the smallest number that could be under [a] is 17.
Moving left 15 units then right 25 units is the same as islolieistalsimislel=2s
moving right 10 units. So, Grogg’s number is 10 units to
the right of Ms. Q’s number.
16 17
ormore or more

The largest number that could be under [¢|is 24.


Ms.Q Grogg So, the largest number that could be under [m] is 23.
Moving left 35 units then right 45 units is the same as SsLelsialLelsials(e les
moving right 10 units. So, Winnie’s number is 10 units to
the right of Grogg’s number.
+10 +10 16 17 23 24
ormore or more orless orless

50 6O 70 So, the number under must be more than 17 and


h
less than 23. Only the whole numbers from 18 to 22 are
Grogg = Winnie
more than 17 and less than 23:
So, Winnie marks 70. 18, 19, 20, 21, 22.

Beast Academy Practice 2A Comparing Chapter 2 Solutions


105. There are 9 different two-digit numbers we can make
using the digits 1, 3, and 6. We label these numbers on
the number line below.

AFOFO HOR HEE EEEEERE HHOIO ROH EEpeat tatatateltlatabecbah bE OPOre rte

1113 16 3133 36
irae
61 63 66

We can rule out any number in the 10’s or 60’s as the


middle number, so the middle number is in the 30’s.
This only works if the smallest number is in the 10’s, and
the largest number is in the 60’s.
So, we have

[3|[_] is halfway between [1] and [6l[_].


Using 1’s, 3’s, and 6’s, there is only one way to fill the
ones digits of these numbers to make a true statement:

(3][6] is halfway between (4][1] and (6][1].


Check:
The distance between 11 and 36 is 25 units.
The distance between 36 and 61 is 25 units. W

106.
een ee ee Ea
7I7I\<(elel<E)* IE)<2I[S]<s]e] x
Sl7)<(6l6)<(7I5]v —[2[3)<(2I[9}<(3]3] v
es aN
fal<fjel<a2)* I2)<(5\5)<2)2] «
H)<f2\<(2|2}v — {2l[2)<(2I5}<(5]2] v

Comparing Chapter 2 Solutions Beast Academy Practice 2A


‘er : ade Solutions

23. 223+195=300+ 110+


8 =418.
24. 528+345
= 800+ 60 + 13 =873.
1. Adding 32+56 gives 3+5 =8 tens and 2+6<8 ones. 25. 393+461 = 700+ 150 + 4 =854.
8 tens is 80, so 8 tens and 8 ones is 80+8 =88. 26. 315+167=400+ 70 + 12 =482
32+56= 80+ 8 = 88. 27. 248+590=700+ 130 + 8 =838.
2. Adding 13+51 gives 1+5=6 tens and 3+1 =4 ones. 28. 273+64
200+ 130 +7==337.
6 tens is 60, so 6 tens and 4 ones is 60+4 = 64. 29. 883+109=900+ 80 + 12 =992.
13+51= 60+ 4 = 64. = 800+ 90
30. 555+346 + 11 =901.
3. Adding 62+34 gives 6+3=9 tens and 2+4 =6 ones.
9 tens is 90, so 9 tens and 6 ones is 90+6 = 96.

62+34= 90+ 6 = 96. 31.


fale[= fax
4. Adding 54+21 gives 5+2=7 tens and 4+1 =5 ones.
fe &
7 tens is 70, so 7 tens and 5 ones is 70+5=75.
fs]+ [2]= [ar
544+21=
70+ 5 = 75.
---
5. Adding 217+461 gives 2+4=6 hundreds, 1+6=7 tens, | [a]=Lo
fal [55]=[re
and 7+1=8 ones. 6 hundreds, 7 tens, and 8 ones is
600+70+8=678. ict

217+461= 600
+ 70 +_8 = 678. a
ar]+ [20|= [ar
---
6. Adding 47+32 gives 4+3=7 tens and 7+2=9 ones.
7 tens and 9 ones is 79.
7. Adding 61+25 gives 6+2=8
8 tens and 6 ones is 86.
tens and 1+5=6 ones.
rr« [ss=fs
8. Adding 136+53 gives 1 hundred, 3+5 =8 tens, and
6+3=9 ones. 1 hundred, 8 tens, and 9 ones is 189.

9. Adding 64+315 gives 3 hundreds, 6+ 1=7 tens, and


4+5=9 ones. 3 hundreds, 7 tens, and 9 ones is 379.

10. Adding 250+37 gives 2 hundreds, 5+3=8 tens, and


0+7=7 ones. 2 hundreds, 8 tens, and 7 ones is 287.

11. Adding 270+604 gives 2+6 =8 hundreds, 7+0=7 tens,


and 0+4=4 ones. 8 hundreds, 7 tens, and 4 ones is 874.

12. Adding 316+260 gives 3+2=5 hundreds, 1+6=7 tens,


and 6+0=6 ones. 5 hundreds, 7 tens, and 6 ones is 576.
13. 43+83=120+
6 =126
14. 72+46=110+
8 =118.
15. 54+19=
60 +13 = 73.
16. 36+28= 50+14 = 64.
4

17. 71+83=150+
4 =154.
18. 59+39= 80+ 18 = 98.
19. 52+81=130+_3
=133.
20. 67+28=
80 + 15 = 95
21. 38+83=110+
11 =121.
22. 85+65=140+ 10 =150.

Beast Academy Practice 2A Addition Chapter 3 Solutions [121]


40. 880 +88 = 968. 54. After 3 cars drive from the upper lot to the lower lot, there
are 203-3 = 200 cars in the upper lot, and 277+3 =280
PORN
a a cars in the lower lot.
<.->.-——$$—$S—$
oer
So, there are 200+280 = 480 cars in the whole garage.
880 900
This is the same number of cars that were in the garage
41. 350+57 = 407. before the 3 cars moved. So, 203+277 = 480.
55. The top shelf currently holds 93 books. Adding 100 to |
2 an Mh ee, peer eee a number is easier than adding 93. If we move 7 books
ee
350 400 from the bottom shelf to the top shelf, there will be
93+7 = 100 books on the top shelf. So, we move 7 books
42. 175+29 =204.
to make it easier to find the total number of books.

aee eee Adding 100 books to the 78-7=71 books left on the
a a bottom shelf gives 100+ 71 = 171 books.
175 200
56. To add 98+101+101 peas, we take one pea each from
43. 391 +39 = 430. Ada and Parker and give both to Sander. This leaves 100
peas on each plate, for a total of 100+ 100+100 = 300
peas.

Adding 98+ 101+101 by place value gives


44. 193+22=215. 200+90+10 = 300 peas.

193 200 215

57. Adding 38 to a number is the same as adding 40, then


45. 496 + 56 = 552.
taking away 2.
58. Adding 19 to a number is the same as adding 20 , then
ef taking away 1.
496 500 552
59. Adding 93 to a number is the same as adding 100, then
46. 598+ 223 = 821. taking away 7.
42 +221 60. Adding 28 is the same as adding 30, then taking away 2.

<< So, 133+28=[133|+|


30|-[ 2 |=[161].
598 600 821
61. Adding 91 is the same as adding 100, then taking away 9.
So, 216+91 =[2/6|+|100|-|
7|=[307}.
62. Adding 95 is the same as adding 100, then taking away 5.

47. To make adding 39+26 easier, we take 1 from 26 and So, 256+95 = [256|+{100]-| 5 |= [351],
give it to 39. 63. 27 is 3 less than 30. So, the sum of three 27’s is
So, 39+26 =| 40|+| 25|=| 65 | 3+3+3=49 less than the sum of three 30’s.
48. To make adding 290+86 easier, we take 10 from 86 and We have
give it to 290.
27+27+27 =|30|+{30|+|30|-[ 9 |=[ 81 |
290+86 =[300]+| 76|=[376}. 64. 99 is 1 less than 100. So, the sum of five 99’s is 5 less
49. 52+128 =|50|+|130|=[180}. than the sum of five 100’s.
50. 78+96 =|74|+[100]=[174], We have
51. 393+ 128 =[400]+(121]=[521], 99+99+99+99+99=[500|-[ 5 |=(495]
52. 595+237 =[600|+|232|=(832], 65. 298 is 2 less than 300, and 299 is 1 less than 300.
53. After Marie pours 4 ounces from one pitcher into the So, the sum 298+299+300 is 3 less than the sum of |
other, one pitcher holds 96+4 = 100 ounces of water, and 300+300+300 = 900. |
the other holds 96-4 = 92 ounces of water. We have
The total number of ounces in the two pitchers is 298+299+300=[900]|-| 3 |=[897]
100+92 = 192 ounces.
This is the same amount of water that Marie started with,
SO 96+ 96= 192.

[122 Addition Chapter 3 Solutions Beast Academy Practice 2A.


66. 24 is 1 less than 25. 74. We add by place value:
23 is 2 less than 25. 295+ 140 = 300+130+5 = 435.
22 is 3 less than 25.
So, 25+24+23+22 is 14+2+3=6 less than
25+25+25+25 = 100: We take 5 from 140 and give it to 295:
295+ 140 = 300+ 135 = 435.
So, we have
25+244239+22 = [100
100 |-
|-|6 |=|a
94|
To add 140, we add 5, then add 135 more:
67. 9 is 1 less than 10.
295+ 140 = 295+5+135
= 300+ 135 = 435.
19 is 1 less than 20.
29 is 1 less than 30. 75. We add by place value:
39 is 1 less than 40. 713+65 = 700+70+8=778.
So, 9+19+29+39 is 1+1+1+1 =4 less than
10+20+30+40= 100. We take 5 from 713 and give it to 65:
So, we have 713+65 = 708+70=778.
9+19+29+39=|100|-[ ¥ |=| 96 |
68. 190 is 10 less than 200. To add 65, we add 60, then add 5 more:
So, 190+190+190+190 is 10+10+10+10=40 less than 713+65 = 713+60+5 =773+5=778.
200+200+200+200 = 800. 76. We add by place value:
So, we have 135+65 = 100+90+10=200.
190+190+190+190=| 800|-| 40|=[760]
We take 5 from 135 and give it to 65:
135+65 = 130+70 = 200.
1h We add by place value:
69. We add 100, then take away 4:
470+65 = 400+ 130+5 = 535.
328+ 96 = 328+ 100-4 = 428-4
= 424.

To add 65, we add 30, then add 35 more:


We take 4 from 328 and give it to 96:
470+65 = 470+30+35 = 500+35 = 535.
328+ 96 = 324+100 = 424.
70. We add 100, then take away 4:
589+96 = 589+ 100-4 = 689-4 =685. We take 30 from 65 and give it to 470:
470+65 = 500+35 = 535.
78. We add by place value:
We take 4 from 589 and give it to 96:
339+111=400+40+10=450.
589+ 96 = 585+ 100 = 685.
71. Since 2+96 = 98, we have
702+96 = 700+98 = 798. We take 1 from 111 and give it to 339:
339+111=340+110=450.
79. We take 1 from 111 and give it to 299:
We add by place value:
299+111=300+110=410.
702+ 96 = 700+90+8 = 798.
72. We add by place value:
To add 111, we add 1, then add 110 more:
538+ 140 =600+70+8=678.
299+111=299+1+110=300+110=410.
—"Or=—
80. We add by place value:
To add 140, we add 100, then add 40 more:
77+111 = 100+80+8 = 188.
538+ 140 = 538+ 100+40 = 638+40 =678.
73. We add by place value:
To add 111, we add 100, then add 11 more:
1904+ 140 = 200+ 130 = 330.
774111 =77+100+11=177+11 =188.
81. We take 25 from 426 and give it to 75:
To add 140, we add 10, then add 130 more:
75+426 = 100+401 = 501.
190+ 140 = 190+ 10+130 = 200+ 130 = 330.

We add by place value:


We take 10 from 140 and give it to 190: 75+426 = 400+90+11=501.
190+ 140 = 200+ 130 = 330.

Beast Academy Practice 2A Addition Chapter 3 Solutions |123]


82. We add by place value: =114. But, 116+114 =230, not 300.
block is 10+104
57+75=120+12=132.

We take 3 from 75 and give it to 57:


57+75 = 60+72 = 132.
83. We take 20 from 75 and give it to 680:
Since 230 is less than 300, we should try a larger number
680+75 = 700+55 = 755.
in the bottom-middle block. If we fill the bottom-middle
block with 100, the middle-left block is 106+ 100 = 206,
We add by place value: and the middle-right block is 100+ 104 = 204. But,
680+ 75 = 600+ 150+5=755. 206+204 = 410, which is too large.

We can continue increasing or decreasing our guess for


the bottom-middle number until we find the number that
works.

The bottom-middle block gets added to both 106 and 104


to give the numbers in the middle row:
88.
106+[ _]and [_]+104.
We add these two numbers to get 300.
106+{ ]+[__]+104=300.
Since 106+ 104 = 210, the bottom-middle block is the
number we add twice to 210 to get 300.
90. In the middle-right block, we have 23+[ 76 |=99.
210+{_]+{__]=300.
In the bottom-left block, we have [12 ]+til 26:
Since 210+[45]+[45]= 300, the bottom-middle block
In the bottom-right block, we have 11 +[65|= [76|.
is 45. We fill the rest of the pyramid as shown.

91. In the middle-right block, we have 195 +[205]= 400.


In the bottom-middle block, we have [110]+95 =(205|.
In the bottom-left block, we have [85 |+{110]= 195.
93. We add by place value:
34+34 =60+8 =68.
94. We add by place value:
38+38 =60+16=76.

92. There are no blocks that we can fill just by adding, or by We add, then take away: |
finding the missing number in an addition problem. 38+38 = 40+40-4=80-4=76.
So, we try filling in the bottom-middle block and look for a 95. We add by place value:
pattern. If we fill the bottom-middle block with 10, then the 70+70 = 140. |
middle-left block is 106+ 10 = 116, and the middle-right
96. We add by place value: ;
234 +234 = 400+60+8 = 468.

Addition Chapter 3 Solutions Beast Academy Practice 2A |


|
97. We add by place value:
382+ 382 = 600+ 160+4 = 764.
98. We add by place value: 108. The ones digit of 12+22+324+42+52+62+72+482+492 is
390+390 = 600+ 180 = 780. the same as the ones digit of 2+2+2+2+2+2+2+2+2=18.
So, the ones digit of Ralph’s sum is 8.
We add, then take away: 109. For the sum of two numbers to have ones digit 4, the
390+390 = 400+ 400-20 = 780. sum of their ones digits must be 4 or 14. There is not a
99. Adding four 13’s is the same as adding two pairs of 13's. pair of numbers in the list whose ones digits sum to 4.
Each pair of 13’s adds to 13+13= 26. So, we can double The only pair whose ones digits sum to 14 is 246+248,
26 to get the same sum as 13+13+13+13. since 6+8 = 14. We circle 246 and 248.
Check: 13+ 134+13+13=52, and 26+26=52.V
243 244 245 247
100. To add 248+252, we can take 2 from 252 and give it to
110. Since we only care about the ones digit, we can ignore
248. This gives 250+250. So, we can double 250 to get
the 9 tens in 97 and look for the number of 7’s we add to
the same sum as 248+252.
get a sum with ones digit 8.
101. See ee ae a [8 0a| The sum of two 7’s is 7+7 = 14.

102. 2) Te) aeass)Ls] Adding a third 7 gives a sum of 14+7=21.


Adding a fourth 7 gives a sum of 21+7 =28.

103. [5] (ie) [20 [40] [0] [87 (320) [a0 97+97+97+97 has the same ones digit as
7+7+7+7=28, which is 8.

104. [>] Cae] (a8) [6] [ne] (zea)[aaa] [a8 So, 4 is the smallest number of 97’s we can add to get a
result with ones digit 8.
105. [ir] (a) [aa] Cae] [re] [ase] [roa . TO find the ones digit of the sum of 111 copies of 111,
we can ignore the hundreds digits and the tens digits and
106. We work backwards. add all of the ones digits.
First, we find the number that Ralph doubled to get 92. The sum of one hundred eleven 1’s is 111.
40+40=80, so 45+45=90, and 46+46=92. So, the ones digit of the sum of 111 copies of 111 is 1.
So, the number Ralph doubled to get 92 is 46.
112. 2 bananas cost 19+19 =20+20-2 = 38 cents.
Then, we find the number that Ralph doubled to get 46.
5 strawberries cost 9+9+9+9+9 =50-5=45 cents.
Since 23+23 = 46, the number Ralph doubled is 23.
3 mangos cost 99+99+99 = 300-3 = 297 cents.
So, Ralph started with 23.
4 dragonfruits cost 49+49+49+49 =50+50+50+50-4
107. Doubling 88 gives Winnie 88+88 = 176. So, Grogg adds
cents. 50+50+50+50 = 200, and 200-4 = 196 cents.
two 2-digit numbers to get 176. To make one of the
two numbers as small as possible, we make the other 113. The cost of each fruit has ones digit 9.
number as large as possible. So, the cost of 6 fruits has the same ones digit as
The largest 2-digit number is 99. The smallest number 9+9+9+9+9+9. To add six 9’s, we can add six 10’s,
that could be part of Grogg’s sum is the number that can then take away 6. Six 10’s is 60, and 60-6 = 54. So, the
be added to 99 to get 176. cost of 6 fruits has ones digit 4.

99+77 = 176, so the smallest number that could be part The only price with ones digit 4 is 234 cents.
of Grogg’s sum is 77. 233 cents 235 cents 236 cents
For example, 6 apples cost 234 cents.
Starting with 88+88, we can increase one of the 114. The cost of each fruit has ones digit 9.
numbers in the sum and decrease the other by the same Since 9+9= 18, adding 2 fruit prices always gives a
amount without changing the sum of the numbers. result that ends in 8.
For example, 88+88 = 89+87.
Since 9+9+9=27, adding 3 fruit prices always gives a
99 is the largest 2-digit number. So, we add 11 to one result that ends in 7.
number in 88+88, and subtract 11 from the other.
Since 9+9+9+9 = 36, adding 4 fruit prices always gives
88+88 = 99+77.
a result that ends in 6.
So, the smallest number that could be part of Grogg’s
The pattern continues. Each time we add a fruit, the ones
sum is 77.
digit of the total cost decreases by 1.
Since Winnie paid 186 cents, she bought 4 fruits.

Beast Academy Practice 2A Addition Chapter 3 Solutions [125]


Note that 14 fruits also give a total sum with ones digit 6,
but since Winnie did not buy 2 fruits that are the same,
she could not have bought more than 6 fruits. 17, 88 has 5 more tens and 4 more ones than 34.
Winnie either bought a banana, an orange, an apple, So, 34+[
54|=88. |
and a mango, or she bought a strawberry, an orange, a 118. 58 has 4 more tens and 2 more ones than 16. i
dragonfruit, and a mango. So, 16+]
42]=58. |
115. As described in the previous problem, 119: 91 has 6 more tens and 1 more one than 30.
the cost of 1 fruit has ones digit 9, So, 30+| 61 ]=91.
the cost of 2 fruits has ones digit 8,
the cost of 3 fruits has ones digit 7,
120. 70 has 5 more tens than 24. But, adding 5 tens to 24
gives 74, which is too big. Adding 4 tens to 24 gives 64.
the cost of 4 fruits has ones digit 6,
Then, adding 6 ones gives 70.
the cost of 5 fruits has ones digit 5, and so on.
The ones digit counts down from 9 to 0, then the pattern
So, 24+|
46 |=70.
repeats every 10 fruits. So, the costs of 1, or 11, or 21, 121. 93 has 4 more tens than 56. But, adding 4 tens to 56
or 31 fruits all have ones digit 9. The costs of 2, or 12, or gives 96, which is too big. Adding 3 tens to 56 gives 86.
22, or 32 fruits all have ones digit 8. Then, adding 7 ones gives 93.

Since Alex paid 108 cents, which has ones digit 8, the So, 56+|
37 |=93.
number of fruits he bought is 2, or 12, or 22, or some 122. 182 has 3 more tens than 154. But, adding 3 tens to 154
other number ending in 2. gives 184, which is too big. Adding 2 tens to 154 gives
First, we try 2 fruits. 174. Then, adding 8 ones gives 182.
2 dragonfruits cost 49+49 = 98 cents, and So, 154+
28|=182.
2 mangos cost 99+99 = 198 cents. So, 108 cents cannot 123. 478 has 1 more hundred, 6 more tens, and 4 more ones
be the cost of 2 of any fruit. than 314. So, 314+{164]=478.
Next, we try 12 fruits. 124. We can think of 240 as 24 tens, and 160 as 16 tens.
12 strawberries cost 9 cents each. To add twelve 9’s we So, 240 has 8 more tens than 160. Since 8 tens is 80,
can add 12 tens and take away 12 ones. 12 tens is 120, 160+]
80|=240.
and 120-12=108. So, Alex bought 12 strawberries. 125. 518 has 2 more hundreds than 333. But, adding
Since strawberries are the cheapest fruit, Alex could not 2 hundreds to 333 gives 533, which is too big.
have bought more than 12 fruits for 108 cents. Adding 1 hundred to 333 gives 433.
116. The ones digit of the price helps us find out how many Then, adding 8 tens to 433 gives 513.
fruits can be bought for 127 cents.
Finally, adding 5 ones gives 518.
The cost of 1 fruit has ones digit 9,
All together, we added 1 hundred, 8 tens, and 5 ones.
the cost of 2 fruits has ones digit 8,
the cost of 3 fruits has ones digit 7, So, 333+[185]=518.
the cost of 4 fruits has ones digit 6,
the cost of 5 fruits has ones digit 5, and so on.
The pattern repeats every 10 fruits.
126. To the right of 23 and 38 is 23+38=61.
Since 127 cents has ones digit 7, the number of fruits is
To the right of 38 and 61 is 38+61 =99.
3, or 13, or 23, or some other number ending in 3.
To the right of 61 and 99 is 61+99 = 160.
To find the largest number of fruits that can be bought,
we look at the cheapest fruit, strawberries. To the right of 99 and 160 is 99+160 = 259.

23 strawberries cost more than 127 cents, so we cannot


buy 23 fruits for 127 cents. CH5)23)(38)(61)(
99){60)-@58)
13 strawberries cost 117 cents, which is 10 cents less Since 8+15 = 23, the number left of 15 is 8.
than 127 cents.

Since a banana costs 10 cents more than a strawberry, if (8(15)(28)(28)(61)-{98)(60)


059)
we replace a strawberry with a banana, we will still have
13 fruits, but the cost will be 117+10 = 127 cents.
So, the largest number of fruits we can buy for exactly
127 cents is 13 (12 strawberries and 1 banana).

Addition Chapter 3 Solutions Beast Academy Practice 2A |


127. To the right of 81 and 131 is 814131 =212. 132. Each number ends in either 4 or 6. To get pairs whose
To the right of 131 and 212 is 131+212 = 343. sum ends in 0, we must pair each number ending in 4
with a number ending in 6.
To the right of 343 and 555 is 343+555 = 898.
First, we look for numbers that can be paired with only
C6
1)631) 019)849}658)(698) one other number. For example, 46 cannot be paired
with 86 or 26, so it must be paired with 54.
Since 50+81 = 131, the number left of 81 is 50.
84 cannot be paired with 14, 24, or 64, so it must be
Since 31+50 = 81, the number left of 50 is 31. paired with 16.

(21){50)(81431)@12)G49}-58)(698)
128. Since 40+65 = 105, the number left of 105 is 65.

COCs
0 878)
To the right of 65 and 105 is 65+105 = 170.
To the right of 170 and 275 is 170+275= 445.

C49)
(68)08)070}-079)448
Since 25+40=65, the number left of 40 is 25.
Since 15+25 = 40, the number left of 25 is 15.

(15){25)(40)(65)4105)170}-G78)48) 133. 40+39+21 = 100.


21+51+28= 100.
(21)
129. Since 201+323 = 524, the number left of 524 is 323. 40+32+28 =100.
OOOC
eo) e23-62)
To the right of 323 and 524 is 323+524 = 847.
134. 65+22+13= 100. Gy
OOO
009)623628) 49) 13+73+14= 100.
65+21+14=100.
Since 122+201 = 323, the number left of 201 is 122.
Since 79+122 = 201, the number left of 122 is 79.
Since 43+79 = 122, the number left of 79 is 43.
135. The right side of the triangle includes two given numbers,
Since 36+43 = 79, the number left of 43 is 36. so we start there.

(a8)}(43(78)(123}{201)923)624)(047) 62+5+33= 100.


10+28+62= 100.
10+57+33= 100.

130. We circle the eight pairs as shown below.


136. The bottom of the triangle includes two given numbers,
so we start there.
49+32+19=100. (10)
49+41+10= 100. (a) 2
10+71+19=
100.
131. We only need to look at the ones digits to find pairs
whose sum ends in zero.
For example, since 4+6 = 10, we know 24+46 ends in 0. 137. The right side of the triangle includes two given numbers,
We circle the eight pairs as shown below. so we start there.
9+70+21 = 100.
3+88+9 = 100.
3+76+21 = 100.

£2,

Beast Academy Practice 2A Addition Chapter 3 Solutions


138. None of the sides include two given numbers, so it’s hard 143. 49+36+51 144. 38+384+32+32
to know where to begin. be $ Wee
We try placing a few different numbers at the top of the
triangle, then use these numbers to complete the left and
right sides so that they have a sum of 100.

145.
80+30+70=180 70+30+60=160 60+30+50=140
None of these work, because the sum of the bottom row
is 180, 160, or 140, not 100. But, we are getting closer!
Increasing the value in the top circle decreases the
values in the bottom two circles. We increase the value
of the top circle until the sum of the bottom row is 100.
146. 58+20+42+39+80
a

Nr eae

Whatever number we put in the top circle, the number in


the bottom-left circle is always 10 more than the number 147. 79+21+85+15+56
in the bottom-right circle. Mian eh
The sum of the three numbers in the bottom row is 100.
So, the two missing numbers in the bottom row must
have a sum of 70.
30 and 40 are the only two numbers that sum to 70
where one number is 10 more than the other. We use
these numbers in the bottom row, and fill the top circle 148. 119+226+431+74
with 50 as shown. Sa
+{300|

149. 16+34 = 50. We can pair each 16 with a 34 to make one 50.
Then, we add five 50’s to get 250.

164+34+16+34+16+34+16+34+16+34
139. 38+91+9
WA
140. ae27/seis
Se
wg
EA
50 + 50 + 50 + 50 +
sla
50

Awa neg

150. We can make three pairs that sum to 40. Adding three
40’s gives 120.
141. 16+33+14+7 142. 39+12+38
Nes
VY a 15+17+19+21+23+25

\
a
40 + 40 + 40
Sea

|
: |

Addition Chapter 3 Solutions Beast Academy Practice 2A _


151. Adding 1 through 19 is easiest if we pair numbers whose sum ends in 6. For example, if the first number we circle
sum is 20. We can make 9 of these pairs. has ones digit 1, the other number must have ones digit
1+19=20 6+14=20 5, since 1+5=6.

2+18=20 7+13=20 We find 451+345 = 796.

OHalA= 20 8+12=20 123@5)2G45


4+16=20 9+11=20 159. The only digits in the row are 1’s and 4’s.

5+15=20 The hundreds digit of the sum is 5. So, one number has
hundreds digit 4, and the other has hundreds digit 1.
10 is the only number we didn’t pair with another number.
So, the sum of every whole number from 1 to 19 is the The tens digit of the sum is 5. So, one number has tens
same as the sum of nine 20’s and one 10. digit 4, and the other has tens digit 1.

20+20+20+20+20+20+20+20+20+10 The ones digit of the sum is 8. So, the ones digits of the
numbers we circle must both be 4’s.
We add these to get 190
We find 414+144 =558.
20+20+20+20+20+20+20+20+20+10

Two numbers that sum to 850 have ones digits that sum
= 100 + SOMO 160.
to 0 or 10. Since none of the available digits sum to 0, we
look for ones digits that sum to 10.
The only digits in the row that sum to 10 are 7 and 3, or
5 and 5. We cannot circle two numbers with ones digit 5
without overlapping. So, the ones digits must be 7 and 3.
We find 117+733 = 850.

152. Two numbers that sum to 70 have ones digits whose 161
sum ends in 0.
We cannot use hundreds digit 7 or 8 in a pair of 3-digit
numbers whose sum is 800. We look for a pair of 3-digit
2 and 8 are the only digits in the row whose sum ends numbers with hundreds digits that are less than 7.
in 0. So, these must be the ones digits of the numbers
We find 577+223 = 800.
we circle. We find 22+48 =70.
11@34@88
153. If both numbers are less than 50, their sum will be less
than 50+50 = 100. So, one number must be at least 50.

55 and 56 are the only 2-digit numbers we can circle that


are at least 50. So, we must circle either 55 or 56. 162. 163.
We find 44+56 = 100.
2334 456 6
154. We find 6544321 =975.

155. We find 233 +556 = 789. 164. 165.


1@33465677899
156. The only digits in the row are 2’s and 1’s.
Since 432 has ones digit 2, the numbers we circle must
both have ones digit 1.
We find 211+221 = 432.
166. 167.
12@1 G2
157. The only digits in the row are 7’s and 3's.
The numbers we circle must both have hundreds digit 3.
Otherwise, their sum will be too large.
We find 373+337 = 710.

158. Two numbers that sum to 796 have ones digits whose

Beast Academy Practice 2A Addition Chapter 3 Solutions


177. We could add five copies of 222, then look for the
number we double to get that sum. Since the sum of five
222’s is very large, we look for a better way.
To split five 222’s into two equal groups, we can put two
222’s in each group. Then, we can split the remaining
222 into two equal parts. Since 111+111=222, we can
split 222 into two 111’s.
222 4+2224+222+222+222

444+1114+111+444

=
Seabees
55 ete OO

So, 222+222+111=555 is the number we double to get


the sum of five 222’s.
— or—

We can split each 222 into 111+111. This gives us


the sum of ten 111’s. We can make two groups of
11141114+11141114+111=555.

228s" 222 222) 222 et ace

A444 414499449914 191419141194 1904 1114111

= 555 + 555

So, 555 is the number we double to get the sum of five


222’s.
178. The smallest 3-digit number is 100. We create a list of
pairs of numbers that sum to 211, starting with 100+111.
100+111 104+107 108+103
175. Adding nine 79’s is the same as adding nine 80’s, then 101+110 105+106 109+102
taking away nine 1’s. Adding nine 80’s gives a number 102+109 106+105 110+101
with ones digit 0. Subtracting 9 from a number with ones 103+108 107+104 111+100
digit 0 gives a result with ones digit 1. So, there are 12 different ways to add two 3-digit
In fact, the sum of nine 79’s is 711. numbers to get a sum of 211.

176. We can pair all of the numbers that end in 0 to get 179. For a sum of three numbers to have ones digit 3, the
10+20+30+40 = 100. sum of their ones digits must end in 3.

Then, we add the remaining numbers to get The smallest possible sum of three ones digits in the list
11+22+33=66. is 5+6+7=18, and the largest sum is 7+8+9 = 24.
The only number from 18 to 24 with ones digit 3 is 23, so
Finally, 100+66 = 166.
the sum of the ones digits must be 23.
10+11+20+22+30+33+40 The only three ones digits in the list that have a sum of
— — 23 are 6, 8, and 9, since 6+8+9 = 23.
100 + 66 So, Grogg’s numbers are 56, 78, and 89, and Grogg’s
sum is 56+78+89 = 223.
180. To make the sum as small as possible, we start by
making the hundreds digits of all three numbers as small
as possible. We cannot write a 0 in the hundreds place.
So, we use 1, 2, and 3 as the hundreds digits of our three
numbers.

Next, we make the tens digits as small as possible. The


three smallest digits we have not used are 0, 4, and 5.

Addition Chapter 3 Solutions Beast Academy Practice 2A —


We use these as the tens digits of our three numbers.
SO Sete age tos 5.
Finally, we make the ones digits as small as possible.
The three smallest digits we have not used are 6, 7, and 8.
We use these as the ones digits of our three numbers.
1.0 642 4. 7.+:3 5. 8
Adding by place value, the hundreds digits give
100+200+300 = 600, the tens digits give 0+40+50 = 90,
and the ones digits give 6+7+8 = 21. So, the smallest
possible sum of the three numbers is 600+90+21 =711.
Note that you could have arranged the digits differently
and still gotten the same sum. As long as you used
1, 2, and 3 as the hundreds digits, 0, 4, and 5 as the tens
digits, and 6, 7, and 8 as the ones digits, the sum will
always be 711. For example, 148+256+307=711.
181. To make three pairs that have the same sum, the
smallest number must be paired with the biggest number.
Otherwise, the sum of one of the other pairs will always
be bigger. For example, if we pair 18 with 40, then the
pair that includes 43 will always have a greater sum than
18+40=58.
So, we pair 18 and 43. Their sum is 18+43 =61.
The other pairs are 21+40=61 and 25+36=61.
182. Since the sum of each group is the same, the sum of all
six numbers is double the sum of each group. The sum
of all six numbers is
104+144+21+22+30+53 = 150.
75+75 = 150, so the sum of the numbers in each group
is 75.
One group includes 22 and 53, and the other group
includes 10, 14, 21, and 30.

Beast Academy Practice 2A Addition Chapter 3 Solutions [131]


Beast Academy Practice 2A |
Beast Academy Practice 2A
|
1

||
||
Beast Academy Practice 2A |
|
Beast Academy Practice 2A
‘arHede¢ For
Beadditional
astA books, printables, and more, visit
cademy.com
PRACTICE
Beast Academy is a book series and online platform that provides a
comprehensive, rigorous, and engaging math curriculum for aspiring
math beasts ages 8-13.

This is Practice 2A in the Beast Academy level 2 series:

2A 2B 2C 74B)
e Place Value e Subtraction’ ¢ Measurement ¢ Big Numbers
¢ Comparing _ * Expressions _ * Strategies (+&-) Algorithms (+&—)
e Addition * Problem Solving Odds & Evens ¢ Problem Solving

The companion book, Guide 2A, uses an entertaining comic book style to
teach the mathematical concepts found in this book.

Beast Academy Online provides over 800 lessons with more than 15,000
problems and 700 videos, all aligned to the Beast Academy book series.

For more books, printables, and BA Online, visit

BeastAcademy.com
ISBN 978-1-934124-31-4
: 9 >
Art of Problem Solving has been developing resources a
for outstanding math students since 1993. We offer
challenging math books and online courses as well as
specialized courses in contest preparation, olympiad
training, and computer science. [>
LL6-
Learn more at artofproblemsolving.com.
9 "TE19S4 T24314 Ut
UT
RMD
Ml
PNT
TUE
MTC
TM

You might also like